Win up to 100% Scholarship

Register Now

Indian Polity

2022

 

 

Question 1

In the Government of India Act 1919, the functions of Provincial Government were divided into “Reserved” and “Transferred” subjects. Which of the following were treated as “Reserved” subjects?

1. Administration of Justice

2. Local Self-Government

3. Land Revenue

4. Police

Select the correct answer using the code given below:

(a) 1, 2 and 3

(b) 2, 3 and 4

(c) 1, 3 and 4

(d)1, 2 and 4

Ans: c

Sub-Theme: Acts and Regulations under the Crown Rule (1858–1947)

The Government of India Act 1919, also known as Montagu-Chelmsford Reforms came into force in 1921 with the sole purpose to ensure Indians of their representation in the Government. The Act divided the provincial subjects into two parts – transferred and reserved, this dual scheme of governance was known as ‘dyarchy’.

Options 1, 3 and 4 are “Reserved Subjects”: The reserved list includes subjects such as Law and Order, Finance, Land Revenue, Administration of Justice, Police, irrigation etc. and the subjects were to be administered by the governor through his executive council of bureaucrats. All-important subjects were kept in the reserved subjects of the Provincial Executive.

Option 2 is “Transferred Subjects”: The transferred subjects were to be administered by ministers nominated from among the elected members of the legislative council and it included subjects such as education, health, Local Self-Government, industry, agriculture, excise, etc.

Other Important Provisions of Government of India Act (Montagu-Chelmsford Reforms) Act, 1919:

  • Relaxed the central control over the provinces by demarcating and separating the central and provincial subjects.
  • Introduced Bicameralism and Direct Elections in the country.
  • It required 3 out of 6 members in the

Viceroy’s Executive Council to be Indian.

  • It extended the principle of communal representation among the Sikhs, Indian Christians, Anglo-Indians and
  • Establishment of a Public Service
  • Separated the provincial budgets from the Central budget.
NOTE: Earlier question came on ‘Dyarchy’ in 2017 and in every alternate year, in one way or another there is question on acts and policies of colonial times

 

Question 2

If a particular area is brought under the Fifth Schedule of the Constitution of India, which one of the following statements best reflects the consequence of it?

(a) This would prevent the transfer of land of tribal people to non-tribal people.

(b) This would create a local self-governing body in that area.

(c) This would convert that area into the Union

(d) The State having such areas would be declared a Special Category State.

Ans: a

Sub-Theme: Scheduled and Tribal Areas/Articles and Schedules of Indian Constitution

Fifth Schedule:

  • The Fifth Schedule of the Constitution deals with the provisions as to the Administration and Control of Scheduled Areas and Scheduled Tribes. Article 244 (1) of the Constitution defines Scheduled Areas as the areas defined so by the President of India and are mentioned in the 5th schedule of the Constitution.
  • Governors can make regulations for peace & good government of Scheduled Areas after consulting the Tribal Advisory Council (TAC).
  • Such regulations may prohibit or restrict the transfer of land by or among members of the scheduled tribes, regulate the allotment of land to members of the scheduled tribes and regulate the business of moneylending in relation to the scheduled tribes.
  • The provisions regarding the administration and control of Scheduled and Tribal Areas of any state except the four states are mentioned under this schedule: Assam, Meghalaya, Tripura, Mizoram.
NOTE: Earlier questions on ‘Fifth Schedule’ came in 2015 and 2019. Therefore, it is advisable to prepare the theme and sub-theme very holistically.

 

Question 3

With reference to the writs issued by the Courts in India, consider the following statements:

1. Mandamus will not lie against a private organisation unless it is entrusted with a public duty.

2. Mandamus will not lie against a Company even though it may be a Government Company.

3. Any public minded person can be a petitioner to move the Court to obtain the writ of Quo Warranto.

Which of the statements given above are correct?

(a) 1 and 2 only

(b) 2 and 3 only

(c) 1 and 3 only

(d) 1, 2 and 3

Ans: d

Sub-Theme: Writ Type and Scope

  • Statement 1 is correct: The word Mandamus means “we command”. It directs activity and commands issued by the court to public officials asking them to perform. It is issued against any public body, corporation, inferior court, tribunal, or government for the same purpose. Mandamus cannot be issued against a private individual or body.
  • Statement 2 is correct: Mandamus won’t be granted against private individuals and companies that have no public duty to perform. It cannot be issued against a legislature, commanding or preventing it to enact legislation.
  • Statement 3 is correct: The meaning of the word Quo Warranto is “By what authority or warrant”. Quo Warranto is issued by the court to inquire into the legality of the claim of a person to a public office. Unlike the other writs, this can be sought by any interested person and not necessarily by the aggrieved person.

 

Question 4

Consider the following statements: 

1. A bill amending the Constitution requires a prior recommendation of the President of India.

2. When a Constitution Amendment Bill is presented to the President of India, it is obligatory for the President of India to give his/her assent.

3. A Constitution Amendment Bill must be passed by both the Lok Sabha and the Rajya Sabha by a special majority and there is no provision for joint sitting.

Which of the statements given above are correct?

(a) 1 and 2 only             

(b) 2 and 3 only

(c) 1 and 3 only            

 (d) 1, 2 and 3

Ans: (b)

Sub-Theme: Procedure for the Amendment

Statement 1 is incorrect: Prior permission of the President is not required to introduce the bill amending the Constitution in India.

Statement 2 is correct: The 24th constitutional amendment made it obligatory for the President to give his assent, when a Constitution Amendment Bill was presented to him.

Statement 3 is correct: Article 368 of the Indian constitution requires that the constitution of India can be amended by both houses of parliament by a 2/3 majority (special majority). In case of disagreement between both houses, there is no provision to summon a joint session of parliament.

Procedure for the Amendment (Article 368):

Borrowed from Constitution of South Africa
Introduction of bills Amendment bill can be introduced only in either house of Parliament
Who can introduce? Either by a minister or private member.
President’s role in introduction of bill Prior permission of the President is not required to introduce the bill.
Type of Majority needed Special Majority: Majority of the total membership of that house (50%) + by a majority of not less than 2/3 of the members of that house present and voting (2/3 of Present and Voting).
Bill in houses Both the houses need to pass the bill with special majority
 Joint seating (Art. 108) There is no provision for a joint sitting in case of disagreement between the two Houses.
Amending federal provisions Special majority + ratification by the legislatures of half of the states by a simple majority.
Role of President in assenting the bill 24th constitutional amendment: It also amended Article 368 to provide expressly that Parliament has power to amend any provision of the Constitution.

The amendment further made it obligatory for the President to give his

assent, when a Constitution Amendment Bill was presented to him.

Role of state legislature in introducing bill State       legislature       cannot introduce such a bill.

 

NOTE: Earlier in 2013 a question on such a basic concept of Constitutional Amendment came from M. Laxmikanth. Therefore, it is advisable to read M. Laxmikanth very minutely and analyse the PYQs.

 

Question 5     

With reference to Deputy Speaker of Lok Sabha, consider the following statements:

1. As per the Rules of Procedure and Conduct of Business in Lok Sabha, the election of Deputy Speaker shall be held on such date as the Speaker may fix.

2. There is a mandatory provision that the election of a candidate as Deputy Speaker of Lok Sabha shall be from either the principal opposition party or the ruling party.

3. The Deputy Speaker has the same powers as of the Speaker when presiding over the sitting of the House and no appeal lies against his rulings.

4. The well-established parliamentary practice regarding the appointment of Deputy Speaker is that the motion is moved by the Speaker and duty seconded by the Prime Minister.

Which of the statements given above are correct?

(a) 1 and 3 only             

(b) 1, 2 and 3

(c) 3 and 4 only             

(d) 2 and 4 only

Ans: a

Sub-Theme: People’s Representative/Speaker and Deputy Speaker

Statement 1 is correct: The date of election of the Deputy Speaker is fixed by the Speaker. While on the other hand the date of election of the Speaker is fixed by the President.

Statement 2 is incorrect: There is no such mandatory provision that the election of a candidate, as Deputy Speaker of Lok Sabha, shall be from either the principal opposition party or the ruling party. It is only by convention that the position of Deputy Speaker is offered to the opposition party in India.

Statement 3 is correct: In case of the absence of the Speaker, the Deputy Speaker presides over the sessions of the Lok Sabha and conducts the business in the house, during such time he/she enjoys the same power as the Speaker when presiding over the sitting of the House. Statement 4 is incorrect: Regarding the appointment of Deputy Speaker, there is no such parliamentary practice of moving motion by Speaker and duty seconded by the Prime Minister.

NOTE: Be attentive while attempting this type of question here in statement 2 and 4 you can see the use of extreme words such as ‘mandatory’ and over emphasising words such as ‘well established’, so it is a red flag, not all time but most of the time such types of statements are generally wrong. Before marking, at least think and then mark the answer, otherwise you may lose 2 marks in haste!!

 

 

Question 6     

Which of the following is/are the exclusive power(s) of Lok Sabha? 

1. To ratify the declaration of Emergency

2. To pass a motion of no-confidence against the Council of Ministers.

3. To impeach the President of India

Select the correct answer using the code given below:

(a) 1 and 2                      

(b) 2 only

(c) 1 and 3                       

(d) 3 only

Ans: b

Sub-Theme: Exclusive powers of LS &RS

  • Statement 1 is incorrect: The proclamation of Emergency must be approved by both the Houses of Parliament within one month from the date of its issue. If approved by both the Houses  of  Parliament,  the  emergency continues for six months, and can be extended to an indefinite period with an approval of the Parliament for every six months.
  • Statement 2 is correct: No confidence Motion can only be initiated and passed in LS.
  • Statement 3 is incorrect: As per Article 61, when a President is to be impeached for violation of the Constitution, the charges can be initiated by either House of Parliament. The impeachment resolution needs to be passed by a majority of two-thirds of the total membership in each house.

 

 

Question 7

Consider the following statements:

1. The Constitution of India classifies the ministers into four ranks Cabinet Minister, Minister of State with Independent Charge, Minister of State and Deputy Minister.

2. The total number of ministers in the Union Government, including the Prime Minister, shall not exceed 15 percent of the total number of members in the Lok Sabha.

Which of the statements given above is/are correct?

(a) 1 only

(b) 2 only

(c) Both 1 and 2

(d) Neither 1 nor 2

Ans: b

Sub-Theme: Council of Ministers

Statement 1 is incorrect: The constitution mentions the word ‘council of ministers’, however it doesn’t classify ministers into 4 categories. Generally, the Prime Minister and other Ministers are collectively known as the Council of Ministers.

Statement 2 is correct: As per 91st CAA, the total number of Central Council of Ministers/ State Council of Ministers including Prime Minister/Chief Minister should not be more than 15% of total strength of Lok Sabha. It stated that the number of ministers in the state legislature, including the CM, in a state shall not be less than 12. A member who is disqualified on the ground of defection shall also be disqualified to be appointed as a minister.

 

Question 8 

Consider the following statements:

1. Pursuant to the report of N. Sanyal Committee, the Contempt of Courts Act, 1971 was passed.

2. The Constitution of India empowers the Supreme Court and the High Court to punish for contempt of themselves.

3. The Constitution of India defines Civil Contempt and Criminal Contempt.

4. In India, the Parliament is vested with the powers to make laws on Contempt of Court.

Which of the above statements given above is/are correct?

(a) 1 and 2 only

(b) 1, 2 and 4

(c) 3 and 4 only

(d) 3 only

Ans: b

Sub-Theme: Contempt of Court

  • Statement 1 is correct: Pursuant to the recommendations made by the H N Sanyal Committee, the Contempt of Courts Act, 1971 was passed to bring transparency and more clarity in the concept of contempt of court.
  • Statement 2 is correct: The Supreme Court and the High Courts of India have been empowered with the power to penalise Contempt of Court under Articles 129 and 215 of the Constitution of India.
  • Statement 3 is incorrect: The Constitution of India does not define Civil Contempt and/or Criminal Contempt. However, the expression has been defined by the Contempt of Court Act of 1971.
  • Statement 4 is correct: Article 142 (2) of the Indian Constitution states that “subject to the provisions of any law made on this behalf by Parliament” the Supreme Court shall have all and every power to make any order on the punishment of any contempt of itself.
NOTE: Even if you don’t have any idea about the H N Sanyal Committee, you can still reach the answer by deducing the other with your knowledge, which is possible if you have thoroughly read M. Laxmikanth. In Laxmikanth it is clearly mentioned that the Constitution of India does not define Civil Contempt and/or Criminal Contempt. However, the expression has been defined by the Contempt of Court Act of 1971.

Now if you eliminate statement 3 you’ll be left with two options i.e. (a) and (b). Now you know that statement 4 is correct as per Article 142(2), hence it has to be option (b) 1, 2 and 4.

 

Question 9

With reference to India, consider the following statements: 

1. Government law officers and legal firms are recognised as advocates, but corporate lawyers and patent attorneys are excluded from recognition as advocates.

2. Bar Councils have the power to lay down the rules relating to legal education and recognition of law colleges.

Which of the statements given above is/are correct?

(a) 1 only                        

(b) 2 only

(c) Both 1 and 2             

(d) Neither 1 nor 2

Ans: b

Sub-Theme: Functions of Judiciary

  • Statement 1 is incorrect: As per the Advocates Act, 1961 Advocates to be the only recognized class of persons entitled to practice law. Corporate Lawyers, as well as patent attorneys, are too recognized as advocates and there’s no prohibition on their recognition as advocates.
  • Statement 2  is  correct: To  recognize universities that offer a law degree as a prerequisite for becoming an advocate and inspect and visit universities, or to direct the State Bar Councils to do so are one of the functions of BCI.

 

Question 10

Consider the following statements: 

1. Attorney General of India and Solicitor General of India are the only officers of the Government who are allowed to participate in the meetings of the Parliament of India.

2. According to the Constitution of India, the Attorney General of India submits his resignation when the Government which appointed him

Which of the statements given above is/are correct?

(a) 1 only                        

(b) 2 only

(c) Both 1 and 2            

  (d) Neither 1 nor 2

Ans: d 

Sub-Theme: Attorney General

  • Statement 1 is incorrect: Attorney General has the right to speak and to take part in the proceedings of Parliament or their joint sitting and any committee of the Parliament of which he is a member, but without a right to vote. He enjoys all the privileges and immunities that are available to a Member of Parliament. While Solicitor General of India and Additional Solicitor General of India have no such rights or privilege.
  • Statement 2 is incorrect: The term of office of the AG is not fixed by the Constitution and also, the Constitution does not contain the procedure and grounds for his removal. He holds office during the pleasure of the president and conventionally, he resigns when CoM resigns or is replaced, as he is appointed on its advice.

 

Question 11

With reference to anti-defection law in India, consider the following statements:

1. The law specifies that a nominated legislator cannot join any political party within six months of being appointed to the House.

2. The law does not provide any timeframe within which the presiding officer has to decide a defection case.

Which of the statements given above is/are correct?

(a) 1 only

(b) 2 only

(c) Both 1 and 2

(d) Neither 1 nor 2

Ans: b

Sub-Theme: Provisions of the Anti-Defection Law

  • Statement 1 is incorrect: A nominated member of a House becomes disqualified for being a member of the House if he joins any political party after the expiry of six months from the date on which he takes his seat in the House. This means that he may join any political party within six months of taking his seat in the House.
  • Statement 2 is correct: The law does not specify any time period for the Presiding Officer to decide on a disqualification plea under the Tenth Schedule.

 

2021

 

Question 1

Which one of the following best defines the term ‘State?

(a) A community of persons permanently occupying a definite territory independent of external control and possessing an organised government

(b) A politically organized people of a definite territory and possessing an authority to govern them, maintain law and order, protect their natural rights and safeguard their means of sustenance

(c) A number of persons who have been living in a definite territory for a very long time with their own culture, tradition and government

(d) A society permanently living in a definite territory with a central authority, an executive responsible to the central authority and an independent judiciary

Ans: a

Sub-Theme: Basic

The State refers to a political institution that represents a sovereign people organised under one independent government within a definite territory and subject to no outside control. Thus, a community of persons permanently occupying a definite territory independent of external control and possessing an organised government.

 

Question 2

Which one of the following factors constitutes the best safeguard of liberty in a liberal democracy? 

(a) A committed judiciary

(b) Centralization of powers

(c) Elected government

(d) Separation of powers

Ans: d

The doctrine of Separation of Powers entails the division of the legislative, executive, and judicial functions of government among different This demarcation prevents the concentration of excessive power by any branch of the Government. It thus helps to safeguard the liberty and rights of the people in a democracy. In the Directive Principles of State Policy, Article 50 mentions separation of judiciary from the executive.

Additional Information:

  • Committed Judiciary: It means a judiciary committed to the policies of the government or ruling party; thus, it can’t protect the liberty of all people.
  • Centralization of Power: Liberal- democratic theory   assumes   that decentralization promotes democratic participation, especially local self- government.
    • Elected Government: Only having an elected government doesn’t safeguard liberty in liberal democracy. g., Pakistan.


 

Question 3

What is the position of the Right to Property in India?

(a) Legal right available to citizens only

(b) Legal right available to any person

(c) Fundamental Right available to citizens only

(d) Neither Fundamental Right nor legal right

Ans: b

Sub-Theme: Right outside FR/Legal

    • Earlier, the Indian Constitution recognized the ‘Right to Property’ as a fundamental right under Article 19(1)(f) and Article 31. Later, Right to Property as a fundamental right was abolished by the 44th Constitutional Amendment Act, 1978 and made it a legal right under Article 300-A.
    • Article 300-A of the Constitutions states that “No person shall be deprived of his property save by Authority of Law”.
    • Thus, Right to Property in India is a legal right available to any person.

 

 

Question 4

Which one of the following in Indian polity is an essential feature that indicates that it is federal in character? 

(a) The independence of the judiciary is  safeguarded.

(b) The Union Legislature has elected representatives from constituent units.

(c) The Union Cabinet can have elected representatives from regional parties.

(d) The Fundamental Rights are enforceable by Courts of Law.

Ans: a

Sub-Theme: Forms of Government

The Indian Constitution adopts the federal system of Government, but with more tilt towards a unitary system of government. So, every state government does not have power of its own. Thus, it is considered as a quasi-federal system. The features of a federation are:

  • Two Government, division of powers,
  • Written Constitution,
  • Supremacy of Constitution,
  • Rigidity of Constitution,
  • Independent Judiciary and 
  • Bicameralism

 

 

Question 5

With reference to India, consider the following statements:

1. There is only one citizenship and one domicile.

2. A citizen by birth only can become the Head of State.

3. A foreigner once granted citizenship cannot be deprived of it under any circumstances.

Which of the statements given above is/are correct?

(a) 1 only                        

(b) 2 only

(c) 1 and 3

(d) 2 and 3

Ans: a

Sub-Theme: Acquisition of Citizenship

Statement  1  is  correct: The Indian Constitution has the provision for single citizenship only, i.e., the Indian citizenship. India’s citizens only owe allegiance to the Indian Union. Additionally, the citizenship of an Indian citizen immediately expires when he or she voluntarily obtains the citizenship of another nation. In India, only one domicile is permitted. A single State or UT may only issue a Domicile Certificate.

  • Statement 2 is incorrect: In India, the Head of the State is the President and as per the Indian constitution both a citizen by birth as well as a naturalized citizen are eligible for the office of President. Thus, a naturalized citizen can also become the Head of the State in India.
  • Statement 3 is incorrect: A foreigner who has previously received Indian citizenship, can be deprived of his citizenship. Indian citizenship can be terminated by the Central government, if:
  • obtained the citizenship by
  • shown disloyalty to the Constitution of India
  • unlawfully exchanged goods or information with the enemy while at war.
  • two years of imprisonment in any country within five years of registration or
  • been ordinarily resident out of India for seven years continuously.

 

Question 6

Under the Indian Constitution, concentration of wealth violates

(a) the Right to Equality

(b) the Directive Principles of State Policy

(c) the Right to Freedom

(d) the Concept of Welfare

Ans: b

Sub-Theme: Socialist Principles

Option (b) is correct: Part IV of the Constitution of India, spanning article 36 to 51, deals with the Directive Principles of State Policy. Article 39 of Directive Principles of State Policy (DPSP) clearly says that the operation of the economic system does not result in the concentration of wealth and means of production to the common detriment.

Additional Information:

Right to Equality: Articles 14 to 18 deals with ‘Right to Equality’ –

    • Equality before Law (Article 14)
    • Prohibition of discrimination based on grounds of religion, race, caste etc. (Article 15)
    • Equality of opportunity in matters of public employment (Article 16)
    • Abolition of Untouchability (Article 17)
    • Abolition of titles (Article 18)

Right to Freedom: Article 19 – 21 deals with ‘Right to Freedom’ – Protection of six rights regarding freedom of: (Article 19)

  • Speech and Expression
  • Assembly
  • Association
  • Movement
  • Residence
  • Profession
    • Protection in respect of conviction for offences (Article 20)
    • Protection of life and personal liberty (Article 21)
    • Concept of Welfare: It itself is a part of

 

Question 7

What was the exact constitutional status of India on 26th January, 1950?

(a) A democratic Republic

(b) A Sovereign Democratic Republic

(c) A Sovereign Secular Democratic Republic

(d) A Sovereign Socialist Secular Democratic Republic

Ans: b

Sub-Theme: Key Words in the Preamble

  • The Preamble to the Indian Constitution is a brief introductory statement that reflects the hopes and aspirations of the people.
  • The ideals behind the Preamble to Indian Constitution is based on ‘Objective Resolution’ which were drafted by Jawaharlal Nehru and adopted by the Constituent Assembly on January 22,
  • The original Preamble, adopted by the Constituent Assembly in 1949, declared India a “Sovereign Democratic Republic”. Later, it was amended by the 42nd Constitutional Amendment Act (1976), and added three new words– “Socialist, Secular and Integrity”.

 

Question 8

Right to Privacy’ is protected under which Article of the Constitution of India? 

(a) Article 15                  

(b) Article 19

(c) Article 21                  

(d) Article 29

Ans: c

Sub-Theme: Right to Privacy

Article 21 of the Indian Constitution deals with the protection of life and personal liberty. In the KS Puttaswamy Case (2017), the Supreme Court held that privacy is a natural right that inheres in all natural persons, and that the right may be restricted only by state action that passes each of the three tests:

  • Such state action must have a legislative
  • It must be pursuing a legitimate state purpose; and
  • It must be proportionate.
NOTE: There are very much-repeated questions around and about Right to Privacy (among other provisions from FR as well) in one way or another. It is advisable to read Fundamental Rights very carefully and from a broader perspective.

 

Question 9

A legislation which confers on the executive or administrative authority an unguided and uncontrolled discretionary power in the matter of application of law violates one of the following Articles of the Constitution of India? 

(a) Article 14                  

(b) Article 28

(c) Article 32                  

(d) Article 44

Ans: a

Sub-Theme: Rule of Law

A legislation which confers on the executive or administrative authority an unguided and uncontrolled discretionary power in the matter of application of law violates Article 14 of the constitution.

The Article 14 of the Indian Constitution denotes that the State shall not deny to any person equality before the law or the equal protection of the laws within the territory of India. The concept of ‘equality before law’ is an element of the concept of ‘Rule of Law’.

Rule of Law:

  • The concept of ‘equality before law’ as enshrined in Article 14 of Indian Constitution is taken from the concept of ‘Rule of Law’ propounded by V. Dicey.
  • Rule of law is a legal principle that states that a country should be ruled by the law rather than by the arbitrary actions of particular government officials.
  • Rule of law is part of the basic structure of the constitution.

The ‘Rule of Law’ has 3 fundamental principles:

  • Absence of arbitrary power
  • Equality before law
  • Primacy of individual rights

From the above concept we came to know that ‘equality before law’ under Article 14 connotes the absence of arbitrary power with the authorities. Therefore, when a legislation confers uncontrolled discretionary powers on any authority, it tends to violate Article 14 of the constitution.

 

Question 10

Constitutional government means: 

(a) a representative government of a nation with federal structure

(b) a government whose Head enjoys nominal powers

(c) a government whose Head enjoys real powers

(d) a government limited by the terms of the Constitution

Ans: d

Sub-Theme: System of Government

The Constitutional Government is defined by the existence of a Constitution The idea behind constitutionalism is to place restrictions on the authority of state so that it does not become autocratic and also to protect the interest of individual liberty. In India the powers of the government are limited by means of the Fundamental Rights enshrined in the Part III of the Indian Constitution, which are essentially given to us against the state actions.

NOTE: Also asked in 2014 and 2020, therefore please do refer to the earlier solution/ explanations.

 

Question 11

We adopted parliamentary democracy based on the British model, but how does our model differ from that model? 

1. As regards legislation, the British Parliament is supreme or sovereign but in India, the power of the Parliament to legislate is limited.

2. In India, matters related to the constitutionality of the Amendment of an Act of the Parliament are referred to the Constitution Bench by the Supreme Court.

Select the correct answer using the code given below.

(a) 1 only                        

(b) 2 only

(c) Both 1 and 2             

(d) Neither 1 nor 2

Ans: (c)

Sub-Theme: System of Government

Statement 1 is correct: Indian Parliamentary system is based on constitutional supremacy, hence power of the Parliament to legislate is limited. Whereas the British Model is based on Parliamentary sovereignty. So, as regards the legislation, the British Parliament is supreme.

Statement 2 is correct: According to the Constitution, Parliament and the state legislatures in India have the power to make laws within their respective jurisdictions. This power is not absolute in nature. The Constitution vests in the judiciary, the power to adjudicate upon the constitutional validity of all laws.

Additional Information:

DIFFERENCE BETWEEN INDIAN & BRITISH MODELS

Indian Model British Model
•   Republican system: Head of State

(President) is indirectly elected.

•   Monarchical system: Head of State (King/Queen) is

hereditary.

 

•   Constitutional supremacy

•   The PM may be a member of any of the houses.

•   A person who is not a member of any house can be appointed as minister, but only for six months.

•   No system of legal responsibility of ministers. Ministers are not required to countersign the official acts.

•   No Shadow Cabinet

•   Parliamentary sovereignty

•   The PM should be a member of the Lower House.

•   Usually, members of parliament are appointed as ministers.

•   System of legal responsibility of the minister. Ministers are required to countersign the official acts.

•   Shadow Cabinet by opposition: to balance the ruling cabinet and prepare its members for future ministerial office.

 

Question 12

Consider the following statements: 

1. In India, there is no law restricting the candidates from contesting in one Lok Sabha election from three constituencies.

2. In the 1991 Lok Sabha Election, Shri Devi Lal contested from three Lok Sabha constituencies.

3. As per the existing rules, if a candidate contests in one Lok Sabha election from many constituencies, his/her party should bear the cost of bye-elections to the constituencies vacated by him/her in the event of him/her winning in all the constituencies.

Which of the statements given above is/are correct?

(a) 1 only                        

(b) 2 only

(c) 1 and 3                       

(d) 2 and 3

Ans: b       

Sub-Theme: People’s Representation

This Question was cancelled by UPSC. Probable Solution: (b)

  • Statement 1 is incorrect: The amendment of RPA in 1996 restricted the number of seats one candidate could contest in one election to two constituencies. And whenever they have won more than one, the candidates have retained only one, forcing bypolls in the rest.
  • Statement 2 is correct: In 1991 Lok Sabha Election, Shri Devi Lal contested from three Lok Sabha constituencies.
  • Statement 3 is incorrect: There is no such provision. If a candidate is contesting from more than one constituency, he has to lodge a separate return of election expenses for every election which he has contested. In every case, the Election Commission of India (ECI) bears the cost of elections/bye-elections in all the constituencies in Lok Sabha and assembly elections as well.

 

 

Question 13

With reference to Indian judiciary, consider the following statements: 

1. Any retired judge of the Supreme Court of India can be called back to sit and act as a Supreme Court judge by the Chief Justice of India with prior permission of the President of India.

2. A High Court in India has the power to review its own judgement as the Supreme Court does.

Which of the statements given above is/are correct?

(a) 1 only                        

(b) 2 only

(c) Both 1 and 2             

(d) Neither 1 nor 2

Ans: a

Sub-Theme: Constitutional Provisions Pertaining to Supreme Court/Jurisdiction and Powers of High Court

  • Statement 1 is correct: As per Article 128 of the Constitution, the Chief Justice of India may, at any time, with the previous consent of the President, can request a retired judge of the Supreme Court or a retired judge of a high court to sit and act as a Judge of the Supreme Court for a temporary period. Such a judge is entitled to such allowances as the president may determine. He will also enjoy all the jurisdiction, powers and privileges of a judge of the Supreme Court.
  • Statement 2 is incorrect: High Court does not has power to review and correct its own judgement. Thus, it is bound by its previous decision.

 

Question 14

At the national level, which ministry is the nodal agency to ensure effective implementation of the Scheduled Tribes and Other Traditional Forest Dwellers (Recognition of Forest Rights) Act, 2006?

(a) Ministry of Environment, Forest and Climate Change

(b) Ministry of Panchayati Raj

(c) Ministry of Rural Development

(d) Ministry of Tribal Affairs

Ans: d         

Sub-Theme: National Commission for Scheduled Castes, Scheduled Tribes & Backward Classes

Option (d) is correct: The Ministry of Tribal Affairs (MoTA) is the nodal agency for the implementation of the Scheduled Tribes and Other Traditional Forest Dwellers (Recognition of Forest Rights) Act, 2006. The ministry deals with forest and livelihood objectives at the national level.

 

 

Question 15

With reference to the Union Government, consider the following statements: 

1. The Gopalaswamy Iyenger Committee suggested that a minister and a secretary be designated solely for pursuing the subject of administrative reform and promoting it.

2. In 1970, the Department of Personnel was constituted on the recommendation of the Administrative Reforms Commission, 1966, and this was placed under the Prime Minister’s charge.

Which of the statements given above is/ are correct?

(a) 1 only

(b) 2 only

(c) Both 1 and 2

(d) Neither 1 nor 2

Ans: b        

Statement 1 is incorrect: In the report ‘Reorganisation of the Machinery of Central Government’ (1950) Sh. N. Gopalaswamy Ayyangar, suggested the grouping of ministries, improvement in the capabilities of the personnel, and also in the working of the Organisation and Method (O&M) Division. It nowhere mentions that a minister and a secretary be designated solely for pursuing the subject of administrative reform and promoting it. Moreover, it was the 1st ARC that suggested a minister and a secretary be designated solely for pursuing the subject of administrative reform and promoting it.

 Statement 2 is correct: In 1970, on the basis of the recommendations of the Administrative Reforms Commission, the Department of Personnel was set up under the charge of Cabinet Secretariat. In 1985, it was placed under the overall charge of the Prime Minister assisted by a Minister of State.

 

2020

 

Question 1 

One common agreement between Gandhism and Marxism is 

(a) the final goal of a stateless society

(b) class struggle

(c) abolition of private property

(d) economic determinism

Ans: a

Sub-Theme: Major Political Theories

The common agreement between Gandhism and Marxism is the final goal of stateless and classless society. But means to achieve these final goals differ under Gandhism and Marxism. A stateless society refers to a society that lacks formal institutions of government.

Additional Information:

  • Karl Marx: State is an instrument of oppression and an organ of the bourgeoisie that only works for maintaining class dominance. Therefore, he strives for class struggle and revolution which will ultimately result in a stateless society.
  • Mahatma Gandhi: According to Gandhiji, even minimal state will require to use This was against non-violence philosophy. Instead, Gandhiji was in favour of strengthening of local self-governments.

 

 

Question 2

In the context of India, which one of the following is the characteristic appropriate for bureaucracy?

(a) An agency for widening the scope of parliamentary democracy

(b) An agency for strengthening the structure of federalism

(c) An agency for facilitating political stability and economic growth

(d) An Agency for the implementation of public policy

Ans: d

Sub-Theme: Basic Understanding of Permanent Executive

Bureaucracy Bureaucracy is the executive arm of the government, where the elected government employs competent state officials to run after state matters. They are selected by the government through the merit-based process. The major objective of bureaucracy in India has not been Parliamentary democracy’s expansion but rather the implementation of the decisions taken by the executive.

Parliamentary democracy results often in change at the helm of executive, the static and permanent bureaucracy provides continuity of policies and execution rather than political stability.

 

Question 3

Consider the following statements: 

1. The Constitution of India defines its structure in terms of federalism, secularism, fundamental rights and democracy.

2. The Constitution of India provides for ‘Judicial review’ to safeguard the citizens’ liberties and to preserve the ideals on which the Constitution is based.

Which of the statements given above is/are correct?

(a) 1 only                        

(b) 2 only

(c) Both 1 and 2             

(d) Neither 1 nor 2

Ans: d

Sub-Theme: Basic Structure of Indian Constitution Statement 1 is incorrect: Basic Structure encompasses basic  and  core  values  like

federalism, secularism, fundamental rights and democracy which forms bedrock of the Indian Constitution.

  • Basic Structure cannot be amended by parliament under Art 368.
  • Basic Structure Doctrine is a Judicially innovative doctrine, and it was neither defined under Constitution nor it was defined by the Supreme Court or any other court.
  • Various judgments of the Supreme Court form the bedrock of Basic Structure Doctrine like: Kesavananda Bharati Case (24 April 1973) and IR Coelho vs State of Tamil Nadu, 2007.

Statement 2 is incorrect: Article 13 of the Indian Constitution though gives the power of judicial review but the word ‘judicial review‘ is nowhere mentioned. The Union or the States shall not make any law that takes away or abridges any of the fundamental rights, and any law made in contravention of the aforementioned mandate shall, to the extent of the contravention, be void.

 

Question 4

The Preamble to the Constitution of India, is:

(a) part of the Constitution but has no legal effect

(b) not a part of the Constitution and has no legal effect either

(c) a part of the Constitution and has the same legal effect as any other part

(d) a part of the Constitution but has no legal effect independently of other parts

Ans: (d)

Sub-Theme: Preamble as Part of the Constitution

The question as to whether the Preamble is a part of the Constitution leads to three landmark SC judgement and those are –

In the Berubari Union Case (1960):

  • The Preamble explains the overall rationale behind several provisions of the
  • If terms used in articles are ambiguous, some assistance of the preamble could be taken for interpretation.
  • SC ruled that Preamble is not a part of the

Kesavananda Bharati Case (1973):

  • The Preamble is a part of the LIC of India Case (1995):
  • SC again held that the preamble is an integral part of the Constitution, and it can also be amended subject to the condition that no amendment is done to the ‘basic

 

Question 5

Which one of the following categories of Fundamental Rights incorporate protection against untouchability as a form of discrimination?

(a) Right against Exploitation

(b) Right to Freedom

(c) Right to Constitutional Remedies

(d) Right to Equality

Ans: d

Sub-Theme: Right to Equality

Articles 14 to 18 deals with ‘Right to Equality’

  • Equality before Law (Article 14)
  • Prohibition of  discrimination  based on grounds of religion, race, caste etc. (Article 15)
  • Equality of opportunity in matters of public employment (Article 16)
  • Abolition of Untouchability (Article 17)
  • Abolition of titles (Article 18)

Hence, Protection against untouchability as a form of discrimination is a Fundamental Right under Right to Equality.

Additional Information:

  • Article 23 & 24 deals with the ‘Right Against Exploitation’
    • Prohibition of traffic in human beings and forced labour (Article 23)
    • Prohibition of employment of children in factories etc. (Article 24)
  • Article 19 – 21 deals with ‘Right to Freedom’
  • Article 19: Protection of six rights regarding freedom of:
    • Speech and Expression
    • Assembly
    • Association
    • Movement
    • residence &
    • Profession
  • Protection in respect of conviction for offences (Article 20)
  • Protection of life and personal liberty (Article 21)
  • Article 32 deals with the ‘Right to Constitutional Remedies’.
NOTE: Question on Untouchability earlier came in 2017.

 

Question 6

Other than the Fundamental Rights, which of the following parts of the Constitution of India reflect/ reflects the principles and provisions of the  Universal  Declaration of Human Rights (1948)? 

1. Preamble

2. Directive Principles of State Policy

3. Fundamental Duties

Select the correct answer using the code given below:

(a) 1 and 2 only             

(b) 2 only

(c) 1 and 3 only             

(d) 1, 2 and 3

Ans: (d)

Sub-Theme: Universal Declaration of Human Rights

  • Statement 1 is correct: The objectives of Preamble such as Justice (social, economic, and political), Equality and Liberty also reflect the principles of UDHR. The Article 3 of Universal Declaration of Human Rights (UDHR) (1948) says that “Everyone has the right to life, liberty and security of person.”
  • Statement 2 is correct: DPSPs are the principles that aim at providing social and economic justice and set the path towards the welfare state. These DPSP act as an obligation on state and are in consonance with human rights, which has a resemblance with UDHR Article 22. The article 22 of UDHR 1948 says that “Everyone, as a member of society, has the right to social security.”
  • Statement 3 is correct: Fundamental duties (Article 51A) are basically civic and moral obligations of all citizens of India. As of now, there are 11 fundamental duties in India, which are to provide opportunities for education to the child between the age of 6 and 14 years by the parents or guardian.

Additional Information:

UDHR Article 26:

    • Everyone has the right to education. Education shall be free, at least in the elementary and fundamental stages.

UDHR Article 27:

    • Everyone has the right freely to participate in the cultural life of the community, to enjoy the arts and to share in scientific advancement and its benefits.
    • Everyone has the right to the protection of the moral and material interests resulting from any scientific, literary or artistic production of which he is the author.
NOTE: Questions related to UDHR have been asked earlier in 2012. Earlier they gave a list of rights and asked whether they are included in UDHR. Thus, keeping note of PYQs is important.

 

Question 7

Which part of the Constitution of India declares the ideal of a Welfare State? 

(a) Directive Principles of State Policy

(b) Fundamental Rights

(c) Preamble

(d) Seventh Schedule

Ans: a

Sub-Theme: Features of the Directive Principles

  • The Directive Principles of State Policy (DPSP) are mentioned under Articles 36 to 51 of the Indian Constitution.
  • The DPSPs have been borrowed from the Constitution of Ireland. It constitutes a comprehensive socio economic and political programme for a modern democratic State. DPSP embodies the concept of a ‘welfare state’ and not that of a ‘police state’.
  • Welfare state refers to a concept of government in which the state plays a key role in the economic and social well-being of its citizens.
  • Article 38 (1): The State shall strive to promote the welfare of the people by securing and protecting as effectively as it may a social order in which justice, social, economic and political, shall inform all the institutions of the national life.
NOTE: This question is a repeated question from 2015.

 

Question 8

With reference to the provisions contained in Part IV of the Constitution of India, which of the following statements is/are correct? 

1. They shall be enforceable by courts.

2. They shall not be enforceable by any court.

3. The principles laid down in this part are to influence the making of laws by the State.

Select the correct answer using the code given below:

(a) 1only                         

(b) 2 only

(c) 1 and 3 only             

(d) 2 and 3 only

Ans: d

Sub-Theme: Features of the Directive Principles The Provisions contained in the Part IV of the Indian Constitution deals with the Directive

Principles of State Policy (DPSP). Under Part IV, Articles 36 to 51 relates to DPSP.

  • Statement 1 is incorrect: The Constitution lays down certain Directive Principles of State Policy, which is non-justiciable in nature. DPSPs were made non-justiciable on the recommendations of the Tej Bahadur Committee Report.
  • Statement 2 is correct: The DPSPs are not legally enforceable by the courts for their violation.
  • Statement 3 is correct: The ‘Directive Principles of State Policy’ denotes the ideals that the State should keep in mind while formulating policies and enacting laws. These are the constitutional instructions or recommendations to the State in legislative, executive and administrative matters.
NOTE: Question on enforceability of DPSP by court came in 2015, it underlines importance of solving PYQs. Moreover, questions on the same subject matter have been asked in 2017.

 

Question 9

In India, separation of judiciary from the executive is enjoined by

(a) the Preamble of the Constitution

(b) a Directive Principle of State Policy

(c) the Seventh Schedule

(d) the conventional practice

Ans: b 

Sub-Theme: Liberal-Intellectual Principles

    • Part IV of the Constitution of India, spanning article 36 to 51, deals with the Directive Principles of State Policy.
    • In the chapter on the Directive Principles of State Policy, Article 50 mentions separation of judiciary from the
    • This article comes under the Liberal- Intellectual Principles, which represent the ideology of liberalism.
    • The doctrine of Separation of powers entails the division of the legislative, executive, and judicial functions of government among different organs.
    • This distinction prevents any branch of the government from concentrating too much authority. As a result, it supports preserving the liberties and rights of the populace in
NOTE: In respect to India the separation of judiciary from the executive is not a conventional practice. The Preamble and Seventh Schedule of the Constitution have already been explained in previous sections.

 

Question 10

In India, Legal Services Authorities provide free legal services to which of the following types of citizens?

1. Person with an annual income of less than Rs. 1,00,000.

2. Transgender with an annual income of less than Rs. 2,00,000.

3. Member of Other Backward Classes (OBC) with an annual income of less than Rs. 3,00,000.

4. All Senior Citizens

Select the correct answer using the code given below:

(a) 1 and 2 only             

(b) 3 and 4 only

(c) 2 and 3 only             

(d) 1 and 4 only

Ans: a

Sub-Theme: Application part of DPSPs Statement 1 is correct: Free legal services are available to the groups of society that are

listed under Section 12 of the Legal Services Authorities Act, they are:

  • Women and children
  • Members of SC/ST
  • Industrial workmen
  • Victims of mass disaster, violence, flood, drought, earthquake industrial disaster
  • Disabled persons
  • Persons in custody
  • Persons whose annual income does not exceed ₹1 lakh or may be prescribed by the State Government
  • Beggars or those who have fallen victim to human trafficking.
  • Statement 2 is correct: Limit for Transgender Rs. 2,00,000.
  • Statement 3 is incorrect: Though there is a provision for free aid to members of SC and ST community, the Legal Services Authority does not mention Other Backward Classes (OBC) while providing free legal services.
  • Statement 4 is incorrect: The provision of free legal services to Senior Citizens is dependent on the rules framed by the respective state governments who can prescribe income limits. As a result, not all elderly people qualify for free legal assistance.
NOTE: In 2013, UPSC asked about the objectives and functions of NALSA. This time they have asked about criteria for free legal services by NALSA.

 

Question 11

A Parliamentary System of Government is one in which:

(a) all political parties in the Parliament are represented in the Government

(b) the Government is responsible to the Parliament and can be removed by it

(c) the Government is elected by the people and can be removed by them

(d) the Government is chosen by the Parliament but cannot be removed by it before completion of a fixed term

Ans: b

Sub-Theme: System of Government

A parliamentary system of government is one in which the executive is accountable to the electorate through a legislature which in turn is periodically elected by the electorate. This responsibility means that the executive stays in power as long as it enjoys the confidence of the house. India has a parliamentary system of government. The Prime Minister and his Council of Ministers (CoM) are responsible to the Lok Sabha. This is called Collective Responsibility. Additional Information:

  • Parliamentary system functions irrespective of whether all parties have representation in the government.
  • The government in the Parliamentary system is not directly elected by the people but indirectly through their representative in Parliament. Therefore, the instrument to remove the government through public vote is not available in the parliamentary
  • It goes against the fundamental principle of parliamentary democracy which requires an executive responsible to the legislature.
NOTE: Observe thematic approach by UPSC here.

Questions on “parliamentary government theme” have been asked as follows:

•   2013:   principles of parliamentary government

•   2015:   basics of parliamentary government

•   2017:   advantage of parliamentary government

•   2017:   principles cabinet government, also known as parliamentary government

•   2020:   basics of parliamentary government

 

Question 12

A constitutional government by definition is a

(a) government by legislature

(b) popular government

(c) multi-party government

(d) limited government

Ans: d

Sub-Theme: System of Government

The Constitutional Government is defined by the existence of a Constitution The idea behind constitutionalism is to place restrictions on the authority of state so that it does not become autocratic and also to protect the interest of individual liberty. In India the powers of the government are limited by means of the Fundamental Rights enshrined in the Part III of the Indian Constitution, which are essentially given to us against the state actions.

Option (a), (b) and (c) are incorrect: A constitutional government may not have a legislature, could be a bi-party government and need not be a popular government and can instead be a monarchy.

NOTE: Question on the same theme has been asked by UPSC in 2014 – “Definition of a constitutional government”.

 

Question 13

Consider the following statements:

1. According to the Constitution of India a person who is eligible to vote can be made a minister in a State for six months even if he/she is not a member of the Legislature of that State

2. According to the Representation of People Act, 1951, a person convicted of a criminal offence and sentenced to imprisonment for five years is permanently disqualified from contesting an election even after his release from prison

Which of the statements given above is/are correct?

(a) 1 only                        

(b) 2 only

(c) Both 1 and 2             

(d) Neither 1 nor 2

Ans: d

Sub-Theme: People’s Representative

Statement 1 is incorrect:  The Indian Constitution lays down the following four qualifications for a person to be chosen as a member of the state legislature, such as –

  • He must be a citizen of
  • He must take an oath or affirmation before the person the Election Commission has designated for this purpose. In his oath or affirmation, he swears.
  • To bear true faith and allegiance to the Constitution of India
  • To uphold the sovereignty and integrity of India
  • He must be not less than 30 years of age in the case of the legislative council and not less than 25 years of age in the case of the legislative assembly.
  • He must possess other qualifications prescribed by Parliament.

Statement 2 is incorrect: As per Section 8 of RPA, 1951, a person convicted of any offence and sentenced to imprisonment for not less than two years shall be disqualified from the date of such conviction and shall continue to be disqualified for a further period of six years since his release not permanently disqualified.

 

Question 14

Consider the following statements: 

1. The President of India can summon a session of the Parliament at such a place as he/she thinks

2. The Constitution of India provides for three sessions of the Parliament in a year, but it is not mandatory to conduct all three sessions.

3. There is no minimum number of days that the Parliament is required to meet in a year.

Which of the statements given above is/are correct?

(a) 1 only                        

(b) 2 only

(c) 1 and 3 only             

(d) 2 and 3 only

Ans: (c)

Sub-Theme: Sessions of Parliament

  • Statement 1 is correct: As per Article 85(1), the Constitution gives the power to the President to summon each House of Parliament to meet. The gap between two sessions must not exceed 6 months.
  • Statement 2 is incorrect: By convention, Parliament meets for three sessions in a year not by Constitution.
  • Statement 3 is correct: There is no provision of the minimum number of days that the Parliament is required to meet in a year.

 

Question 15

Rajya Sabha has equal powers with Lok Sabha in

(a) the matter of creating new All India Services

(b) amending the Constitution

(c) the removal of the government

(d) making cut motions

Ans: b

Sub-Theme: Rajya Sabha

The Rajya Sabha enjoys equal powers with Lok Sabha in matters like –

  • The impeachment of the President,
  • Removal of the vice-president,
  • Constitutional amendments, and
  • Removal of the judges of the Supreme Court and the High Courts.

Additional Information:

  • Rajya Sabha enjoys exclusive powers in the matter of creating new All India Services.
  • In the matter of removal of government, Rajya Sabha has unequal status with respect to Lok Sabha. The Council of Ministers is responsible only to Lok Sabha and not to the Rajya Sabha. Thus, a no-confidence motion cannot be introduced in the Rajya Sabha.
  • Rajya Sabha has less or no powers in the matters of finance like making cut motions, passing money bill, etc. vis-a-vis Lok Sabha.

 

Question 16

Along with the Budget, the finance minister also places other documents before the Parliament which include “The Macro Economic Framework Statement”. The aforesaid document is presented because this is mandated by: 

(a) Long standing parliamentary convention

(b) Article 112 and Article 110(1) of the Constitution of India

(c) Article 113 of the Constitution of India

(d) Provisions of the Fiscal Responsibility and Budget Management Act, 2003

Ans: d

Sub-Theme: Annual Financial Statement

Fiscal   Responsibility   and   Budget

  • Management (FRBM) Act, 2003: The Macro- Economic Framework Statement, presented to Parliament as per the Fiscal Responsibility and Budget Management Act, 2003. The Framework Statement includes assessments of the GDP growth rate, Central Government fiscal balance, and the economy’s external sector balance.

 

Question 17

With reference to the funds under Members of Parliament Local Area Development Scheme (MPLADS), which of the following statements are correct? 

1. MPLADS funds must be used to create durable sets like physical infrastructure for health, education, etc.

2. A specified portion of each MP fund must benefit SC/ST populations.

3. MPLADS funds are sanctioned on yearly basis and the unused funds cannot be carried forward to the next year.

4. The district authority must inspect at least 10% of all work under implementation every year.

Select the correct answer using the code given below:

(a) 1 and 2 only             

(b) 3 and 4 only

(c) 1, 2 and 3 only        

(d) 1, 2 and 4 only

Ans: d        

Sub-Theme: MPLADS

  • Statement 1 is correct: The objective of the scheme is to enable MPs to recommend works of developmental nature with emphasis on the creation of durable community assets based on the locally felt needs to be taken up in their Constituencies.
  • Statement 2 is correct: M.Ps are to recommend every year, works costing at least 15% of the MPLADS entitlement for the year for areas inhabited by Scheduled Caste population and 7.5% for areas inhabited by S.T. population.
  • Statement 3 is incorrect: The released funds under the MPLAD scheme are non-lapsable i.e., if the money is not utilised, it gets carried to the next year.
  • Statement 4 is correct: The district authority should visit and inspect at least 10% of the works every year. It is also expected to involve the MPs in such activities.

 

2019

 

Question 1

In the context of any country, which one of the following would be considered as part of its social capital?

(a) The proportion of literates in the population

(b) The stock of its buildings, other infrastructure and machines

(c) The size of population in the working age group

(d) The level of mutual trust and harmony in the society

Ans: d

Option (d) is correct: The term social capital was popularised by Robert As per OECD, it can be defined as “networks together with shared norms, values and understandings that facilitate co-operation within or among groups”. It consists of trust, mutual understanding, shared values and behaviour that bind together the members of a community and make cooperative action possible. Such interaction enables people to build communities, to commit themselves to each other, and to knit the social fabric.

Option (a) is incorrect: The proportion of literates in the population and the size of population in the working age group are the examples of human capital.

Option (b) is incorrect: The stock of its buildings, other infrastructure and machines is the example of physical capital.

 

Question 2

Under which Schedule of the Constitution of India can the transfer of tribal land to private parties for mining be declared null and void?

(a) Third Schedule

(b) Fifth Schedule

(c) Ninth Schedule

(d) Twelfth Schedule

Ans: b

Sub-Theme: Articles and Schedules of Indian Constitution

Fifth Schedule:

  • Deals with the provisions as to the Administration and Control of Scheduled Areas and Scheduled Article 244 (1) of the Constitution defines Scheduled Areas as the areas defined so by the President of India and are mentioned in the 5th schedule of the Constitution.
  • In the Samatha v. State of Andhra Pradesh & Ors (1997) judgment the Supreme Court held that the transfer of tribal land to private parties for mining was null and void under the Fifth Schedule.
  • Under the fifth Schedule, the Governors can make regulations for peace & good government of Scheduled Areas after consulting the Tribal Advisory Council (TAC).
  • Such regulations may prohibit or restrict the transfer of land by or among members of the scheduled tribes, regulate the allotment of land to members of the scheduled tribes and regulate the business of moneylending in relation to the scheduled tribes.

Additional Information:

Third Schedule: The third schedule of Indian Constitution is related with the Forms of Oaths or Affirmations for – The Union and the State ministers, The candidates for election to the Parliament and State Legislature, Members of the Parliament and State Legislature, The judges of the SC and HC, CAG.

Ninth Schedule: Provisions as to validation of certain Acts and Regulations of the state legislatures dealing with land reforms and abolition of the zamindari system and of the Parliament dealing with other matters. Twelfth Schedule: This schedule was added by the 74th Amendment Act of 1992 and it specifies the powers, authority and responsibilities of Municipalities.

 

Question 3

The Ninth Schedule was introduced in the Constitution of India during the prime minister ship of:

(a) Jawaharlal Nehru

(b) Lal Bahadur Shastri

(c) Indira Gandhi

(d) Morarji Desai

Ans: a

Sub-Theme: Articles and Schedules of Indian Constitution

Ninth Schedule:

  • The Ninth Schedule of the Indian Constitution was added by  the  1st  Amendment(1951) to protect the laws included in it from judicial scrutiny on the ground of violation of fundamental rights. At this time, Jawaharlal Nehru was the Prime Minister of India. He remained in the office from the year 1947 to 1964.
  • This schedule deals with the provisions as to validation of certain Acts and Regulations of the state legislatures dealing with land reforms and abolition of the zamindari system and of the Parliament dealing with other matters.

Additional Information:

Prime Ministership of the following leaders are as follows:

  • Lal Bahadur Shastri: 1964 – 1966
  • Indira Gandhi: 1966 – 1971 and 1980 – 1984
  • Morarji Desai: 1977 – 1979
NOTE: Question on Ninth Schedule came in 2018.

 

Question 4

In the context of polity, which one of the following would you accept as the most appropriate definition of liberty?

(a) Protection against the tyranny of political rulers

(b) Absence of restraint

(c) Opportunity to do whatever one likes

(d) Opportunity to develop oneself fully

Ans: d

Sub-Theme: Key Words in the Preamble

“Opportunity to develop oneself fully” covers liberty fully, therefore, one may surmise that the purpose of ensuring ‘liberty’ is that one has no obstacles or hindrances to realise one’s full potential.

Additional Information:

  • “Protection against the tyranny of political rulers” is a partial definition and has a connotation more towards fundamental rights rather than liberty.
  • “Absence of restraint” only covers part of the definition of liberty.
  • Liberty does not mean ‘licence’ to do whatever one likes; it is to be enjoyed within the limitations mentioned in the Constitution itself.

 

Question 5

Which Article of the Constitution of India safeguards one’s right to marry the person of one’s choice?

(a) Article 19

(b) Article 21

(c) Article 25

(d) Article 29

Ans: b

Sub-Theme: Right to Privacy

Article 21 of the Indian Constitution deals with the protection of Life and Personal Liberty. This very article safeguards one’s right to marry the person of one’s choice. In the famous Hadiya Case (2017), the Supreme Court held that, “The right to marry a person of one’s choice is integral to Article 21 (right to life and liberty) of the Constitution”.

Additional Information:

  • Article 19: This article is related to the protection of six rights regarding freedom of:
    • Speech and expression
    • Assembly
    • Association
    • Movement
    • Residence
    • Profession
  • Article 25: This article deals with the freedom of conscience and free profession, practice and propagation of religion.
  • Article 29: This article is related to the protection of language, script and culture of minorities.

 

Question 6

Consider the following statements:

1. The 44th Amendment to the Constitution of India introduced an article placing the election of the Prime Minister beyond judicial review.

2. The Supreme Court of India struck down the 99th Amendment to the Constitution of India as being violative of the independence of judiciary.

Which of the statements given above is/are correct?

(a) 1 only

(b) 2 only

(c) Both 1 and 2

(d) Neither 1 nor 2

Ans: b

Sub-Theme: Important and Major Constitutional Amendments

Statement 1 is incorrect: The 44th amendment  of  the  Indian  Constitution was significant as it removed partially the distortions that were introduced into the Constitution by 42nd Amendment, but it had no proposal of an article placing the election of the Prime Minister beyond judicial review. The 39th Amendment of the Constitution of India, enacted on 10 August 1975, placed the election of the President, the Vice President, the Prime Minister and the Speaker of the Lok Sabha beyond the scrutiny of the Indian courts. It was passed during the Emergency of 1975–1977.

Statement 2 is correct: The 99th Constitutional Amendment Act would have established the National Judicial Appointments Commission (NJAC). But, in 2015, a five judge Constitution Bench of SC by 4:1 majority upheld the collegium system and struck down the NJAC as unconstitutional.

In this regard, Justice Khehar stated that the proposed NJAC violated the judiciary’s independence because the expectation from the judiciary is to protect the rights of the citizens of this country, and that this expectation can only be ensured by maintaining absolute isolation and independence from the other organs of government.

 

Question 7

With reference to the Constitution of India, consider the following statements: 

1. No High Court shall have the jurisdiction to declare any central law to be constitutionally invalid.

2. An amendment to the Constitution of India cannot be called into question by the Supreme Court of India.

Which of the statements given above is/are correct?

(a) 1 only

(b) 2 only

(c) Both 1 and 2

(d) Neither 1 nor 2

Ans: d

Sub-Theme: Final Interpreter and Guardian of the Constitution

Statement 1 is incorrect: Though the 42nd Amendment Act of 1976 curtailed the power of HC in judicial review and debarred the HC from considering the constitutional validity of any central law.

However, the 43rd Amendment Act of 1978 restored the original position of the HC and moreover now the HC is considered as the principal civil courts of original jurisdiction in each state and union territory, thus they shall have the jurisdiction to declare any central law to be constitutionally invalid.

Statement 2 is incorrect: The Constitution of India has conferred a very extensive jurisdiction and vast powers on the Supreme Court. As the SC is the final interpreter and guardian of the Constitution and also the guarantor of the fundamental rights of the citizens.

Moreover, in the Kesavananda Bharati case (1973), SC laid down a new doctrine of the ‘basic structure’ of the Constitution. Therefore, any constitutional amendment that is ultra vires or goes against the ‘basic structure’ of the Indian Constitution can be nullified by the SC.

 

Question 8

Consider the following statements:

1. The Parliament (Prevention of Disqualification) Act , 1959 exempts several posts from disqualification on the grounds of ‘Office of Profit’.

2. The above-mentioned Act was amended five times.

3. The term ‘Office of Profit’ is well-defined in the Constitution of India.

Which of the statements given above is/are correct?

(a) 1 and 2 only

(b) 3 only

(c) 2 and 3 only

(d) 1, 2 and 3

Ans: a

Sub-Theme: Qualification and Disqualification of Parliament

Statement 1 is correct: There is no bar on how many offices can be exempted from the purview of the law. In the past the Supreme Court also held that the Parliament (Prevention of Disqualification) Amendment Act, 2006 exempting 55 offices occupied by members of Parliament from disqualification was constitutionally valid.

Statement 2 is correct: Parliament has also enacted the Parliament (Prevention of Disqualification) Act, 1959, which has been amended five times to expand the exempted list.

Statement 3 is incorrect: Though the expression ‘office of profit’ is mentioned in the Articles 102(1)(a) and 191(1)(a) of Constitution, however, it has neither been ‘defined’ in the Indian Constitution nor in the Representation of the People Act, 1951. According to Articles 102(1)(a) and 191(1)(a) of Constitution, legislators (MP or MLA) can be barred from holding office of profit under Central Government or state government as it can put them in position to gain financial benefit.

 

Question 9

In India, which of the following review the independent regulators in sectors like telecommunications, insurance, electricity, ?

1. Ad Hoc Committee set up by the Parliament.

2. Parliamentary Department Related Standing Committee

3. Finance Commission

4. Financial Sector Legislative Reforms Commission

5. NITI Aayog

Select the correct answer using the code given below.

(a) 1 and 2

(b) 1, 3 and 4

(c) 3, 4 and 5

(d) 2 and 5

Ans: a   

Sub-Theme: Parliamentary Committees

  • Parliamentary Committee: The purpose of the parliamentary committees is to study and address a wide range of issues that the legislature cannot directly address owing to their magnitude. They also keep an eye on how the executive branch is operating. There are two types of parliamentary committees: permanent or standing committees and ad hoc committees.
  • Department Related Standing Committee: They are elected or appointed periodically, and they work on a continuous basis to secure more accountability of the Executive to the Parliament, particularly financial accountability. The latter are created on an ad hoc basis as the need arises and they are dissolved after they complete the task assigned to them.
  • Ad hoc committees: These committees are created on an ad hoc basis as the need arises and they are dissolved after they complete the task assigned to them.
  • Both the  Parliamentary  committees are responsible to review the independent regulators in sectors like telecommunications, insurance, electricity, etc.

 

Question 10

Consider the following statements:

1. The motion to impeach a Judge of the Supreme Court of Indian cannot be rejected by the Speaker of the Lok Sabha as per the Judges (Inquiry) Act, 1968.

2. The Constitution of India defines and gives details of what constitutes ‘incapacity and proved misbehaviour’ of the Judges of the Supreme Court of India.

3. The details of the process of impeachment of the Judges of the Supreme Court of India are given in the Judges (Inquiry) Act, 1968.

4. If the motion for the impeachment of a Judge is taken up for voting, the law requires the motion to be backed by each House of the Parliament and supported by a majority of total membership of that House and by not less than two-thirds of total members of that House present and voting.

Which of the statements given above is/are correct?

(a) 1 and 2

(b) 3 only

(c) 3 and 4 only

(d) 1, 3 and 4

Ans: c

Sub-Theme: Removal of Judges

  • Statement 1 is incorrect: As per the Judges (Inquiry) Act, 1968, a removal motion signed by 100 members (Lok Sabha) or 50 members (Rajya Sabha) is to be given to the Speaker/ Chairman. The speaker has the discretion to accept or reject the motion.
  • Statement is 2 incorrect: The Constitution of India does not define and gives details of the expression ‘incapacity and proved misbehaviour’ of the Judges of the Supreme Court of India which are the ground for the impeachment of the Judges.
  • Statement 3 is correct: The details of the process of impeachment of the Judges of the Supreme Court of India is governed by the Judges Enquiry Act (1968).
  • Statement 4 is correct: If the motion for the impeachment of a Judge is taken up for voting, the law requires that the motion be passed by a ‘special majority’ i.e. majority of the total membership of the House and majority of not less than two thirds members present and voting.

 

Question 11

With reference to the Constitution of India, prohibition or limitations or provisions contained in ordinary laws cannot act as prohibitions or limitations on the constitutional powers under Article 142. It could mean which one of the following.

(a) The decisions taken by the Election Commission of India while discharging its duties cannot be challenged in any court of law.

(b) The Supreme Court of India is not constrained in the exercise of its powers by laws made by the Parliament.

(c) In the event of a grave financial crisis in the country, the President of India can declare a Financial Emergency without the counsel from the Cabinet.

(d) State Legislatures cannot make laws on certain matters without the concurrence of the Union Legislature.

Ans: b

Sub-Theme: Jurisdiction and Powers of Supreme Court

Article 142 of the Indian Constitution deals with the Enforcement of decrees and orders of the Supreme Court and orders as to discovery, etc. The SC in the exercise of its jurisdiction may pass such decree or make such order as is necessary for doing complete justice in any cause or matter pending before it. This is often termed as ‘judicial activism’. Also in the Union Carbide Case, 1989, the Supreme Court while overriding the laws made by the Parliament invoked Article 142 to provide relief awarding the compensation to victims affected in the Bhopal Gas Tragedy.

Additional Information:

  • The ECI has the power to administer the election to Lok Sabha, Rajya Sabha, State Legislative Assemblies in India, and the offices of the President and Vice President in the country. The decision of the election commissions can be challenged by the petitions in the High Court and Supreme
  • Article 360 empowers the president to proclaim a financial emergency if he is satisfied that a situation has arisen due to which the financial stability or credit of India or any part of its territory is threatened but the proclamation should be approved by the Parliament through a simple majority within two months.
  • According to Article 196 the state legislature can make laws on State List as well as on Concurrent List.

 

Question 12

With reference to the Legislative Assembly of a State in India, consider the following statements:

1. The Governor makes a customary address to Members of the House at the commencement of the first session of the year.

2. When a State Legislature does not have a rule on a particular matter, it follows the Lok Sabha rule on that matter.

Which of the statements given above is/are correct?

(a) 1 only

(b) 2 only

(c) Both 1 and 2

(d) Neither 1 nor 2

Ans: a      

Sub-Theme: Legislative Powers of Governor

Statement 1 is correct: Article 176(1) states that, through the legislative powers of the Governor, he can address the state legislature at the commencement of the first session after each general election and the first session of each year.

Statement 2 is incorrect: The State legislature is a constitutional body and derives its power from the Constitution itself and as per Article

208 (1) of the Constitution, a house of the Legislature of a State may make rules for regulating subject to the provisions of this Constitution, its procedure and the conduct of its business. This implies that the state legislature does not follow Lok Sabha rules, no

such provision exists.

 

Question 13

Which one of the following suggested that the Governor should be an eminent person from outside the State and should be a detached figure without intense political links or should not have taken part in politics in the recent past?

(a) First Administrative Reforms Commission (1966)

(b) Rajamannar Committee (1969)

(c) Sarkaria Commission (1983)

(d) National Commission to Review the Working of the Constitution (2000)

Ans: c        

Sub-Theme: Centre’s Control Over State Legislation

  • 1st ARC Recommendation (1966): The first ARC recommended the appointment of persons having long experience in public life and administration and non-partisan attitude as governors.
  • Rajamannar Committee (1969): The Rajamannar Committee recommended the provision that the state ministry holds office during the pleasure of the governor should be omitted.
  • Sarkaria Commission (1983): The “Sarkaria Commission” recommended that the Governor should be a distinguished outsider without intense political links, that he should not have recently participated in politics, and that he should not be a member of the ruling party.
  • National Commission to Review the Working of the Constitution (2000): The committee recommended that the president should appoint the governor of a state only after consultation with the chief minister of that state.

 

2018

 

Question 1

Which one of the following reflects the most appropriate relationship between law and liberty?

(a) If there are more laws, there is less liberty.

(b) If there are no laws, there is no liberty.

(c) If there is liberty, laws have to be made by the people.

(d) If laws are changed too often, liberty is in danger.

Ans: b

Sub-Theme: Key Words in the Preamble

  • Where there is no Law, there is no For liberty is to be free from restraint and violence from others, which cannot be where there is no law.
  • If there are no laws, there is no liberty. Liberty:

Liberty can be seen from two perspectives– negative and positive.

  • Negative liberty: It is concerned with the inviolable area of non-interference and not with the conditions in society, i.e. absence of obstacles, barriers or constraints.
  • Positive liberty: Recognizes that one can be free only in society and not outside it, therefore it is the possibility of acting or the fact of acting in such a way as to take control of one’s life and realise one’s fundamental Safeguards of Liberty:
  • Declaration of rights of the individuals: It is an important safeguard of liberty. In this manner, the government can be prevented from interfering with citizens’ liberty.
  • The independent and impartial judiciary is aptly referred to as the watchdog of liberty.
  • Decentralisation of powers: History is witness to the fact that concentration of power has very often led to despotism.
  • Separation of powers, i.e., the executive, the legislature and the judiciary being separate, is a great ally of liberty. Montesquieu said, “Power should be a check on power.”
  • Rule of law or equality in the eyes of the law is also an important safeguard of This is the bulwark against discrimination based on caste, class, colour, creed, etc.

Law:

  • It works as the aggregate of legislation, judicial precedents, and accepted legal principles; the body of authoritative grounds of judicial and administrative action; esp., the body of rules, standards, and principles that the courts of a particular jurisdiction apply in deciding controversies brought before them.
NOTE: John Locke in one of his treatises of civil government wrote- “Where there is no law, there is no freedom”.

 

Question  2 

If the President of India exercises his power as provided under Article 356 of the Constitution of a particular State, then

(a) The Assembly of the State is automatically dissolved.

(b) The powers of the Legislature of that State shall be exercisable by or under the authority of the Parliament.

(c) Article 19 is suspended in that State.

(d) The President can make laws relating to that State.

Ans: b

Sub-Theme: State Emergency/ President’s Rule

  • The proclamation of the President’s Rule is also known as ‘State Emergency’ or ‘Constitutional Emergency’.
  • When the President’s Rule is imposed in a state, the President dismisses the state council of ministers headed by the chief minister and parliament passes the state bills and budgets.
  • Ground for declaration of President’s Rule in a State:
    • Article 356: The president has the authority to declare if he is satisfied that the state government cannot function in accordance with the Constitution’s provisions. (The President can act on the report of the governor or otherwise).
    • Article 365: If a state fails to give effect or comply with any direction from the Centre, then it is lawful for the president to impose president’s rule.

Additional Information:

  • Dissolution of the State Legislative Assembly is not necessarily the consequence of
  • When the President’s Rule is imposed in a state, the President dismisses the state council of ministers headed by the chief minister and parliament passes the state bills and budgets.
  • In case of President’s Rule or State Emergency there is no effect on Fundamental Rights like Article 19.
  • According to the 44th Constitutional Amendment only when National Emergency is declared on the grounds of “war or external aggression” & not the ground of “armed rebellion” the Fundamental Rights like Article 19 gets suspended with effect of Article 358.
  • The Parliament passes the state legislative bills and the state budget during President’s

 

Question  3

With reference to the election of the President of India, consider the following statements: 

1. The value of the vote of each MLA varies from State to State.

2. The value of the vote of MPs of the Lok Sabha is more than the value of the vote of MPs of the Rajya

Which of the statements given above is/are correct?

(a) 1 only

(b) 2 only

(c) Both 1 and 2

(d) Neither 1 nor 2

Ans: a

Sub-Theme: Election of President

  • Statement 1 is correct: The President of India is elected indirectly by the Electoral College and it is conducted in accordance with the system of proportional representation by means of single transferable vote. The President of India is elected by an electoral college consisting of:
    • Elected members of the two Houses of Parliament and Legislative Assemblies of the States
    • States includes national capital territory of Delhi and the Union territory of
    • Do not participate: Nominated members of both the houses of the parliament & of state legislative assemblies and all the members of legislative councils of states.
    • Value of Vote: The value of votes of MLAs would differ from State to State as it is based on population criterion and population of each state has a different value.
  • Statement 2 is incorrect: The value of vote of each MP is the same whether it is Lok Sabha or Rajya Sabha. There shall be uniformity in the scale of representation of the different states at the election of the President as follows:
    • Value of vote of an MLA of a state: (Population of state/Total number of elected members of state legislative assembly) * 100
    • Value of vote of an MP: (Total value of votes of MLA’s of all states/Total number of elected members parliament) * 100

 

Question  4

Consider the following statements:

1. In the first Lok Sabha, the single largest party in the opposition was the Swatantra Party

2. In the Lok Sabha, a “Leader of the Opposition” was recognised for the first time in 1969.

3. In the Lok Sabha, if a party does not have a minimum of 75 members, its leader cannot be recognised as the Leader of the Opposition.

Which of the statements given above is/are correct?

(a) 1 and 3 only

(b) 2 only

(c) 2 and 3 only

(d) 1, 2 and 3

Ans: b

Sub-Theme: Leader of Opposition

  • Statement 1 is incorrect: The Communist Party of India (CPI) was the single largest opposition party in the First general Election. CPI won in 16 seats. The Indian National Congress (INC) won a landslide victory, winning 364 of the 489 seats and 45% of the total votes polled. Jawaharlal Nehru became the first democratically elected PM of the country.
  • Statement 2 is correct: In 1969, an official leader of the opposition was recognized for the first time. However, it was given statutory recognition in 1977.
  • Statement 3 is incorrect: In order to receive the status of opposition party and Consequent Leader of Opposition Tag a party should secure 10% of the total strength of the House i.e., 55 seats in the Lok Sabha.”]

 

Question  5

With reference to the Parliament of India, which of the following Parliamentary Committees scrutinize and report to the House whether the powers to make regulations, rules, sub-rules, by-laws, etc. conferred by the Constitution or delegated by the Parliament are being properly exercised by the Executive within the scope of such delegation?

(a) Committee on Government Assurances

(b) Committee on Subordinate Legislation

(c) Rules Committee

(d) Business Advisory Committee

Ans: b

Sub-Theme: Parliamentary Committees

Committee on Subordinate Legislation:

  • Composition: 15 members from the Lok Sabha are nominated by the Speaker. A Minister is not nominated to this
  • Functions: Scrutinizes and reports to the House whether the powers to make regulations, rules, sub-rules, by-laws etc. conferred by the Constitution or delegated by Parliament are being properly exercised by the executive within the scope of such
NOTE: Simple yet confusing description based question by UPSC. Trick to mark such description based questions correctly is to keep focus on keywords. For instance, in the above question key words rules, sub- rules, by-laws, delegated by the Parliament hints towards option (b) Committee on Subordinate Legislation.

 

Question  6

Regarding Money Bill, which of the following statements is not correct?

(a) A bill shall be deemed to be a Money Bill if it contains only provisions relating to imposition, abolition, remission, alteration or regulation of any tax.

(b) A Money Bill has provisions for the custody of the Consolidated Fund of India or the Contingency Fund of India.

(c) A Money Bill is concerned with the appropriation of money out of the Contingency Fund of India.

(d) A Money Bill deals with the regulation of borrowing of money or giving of any guarantee by the Government of India.

Ans: c

Sub-Theme: Money Bill

Money Bill (Article 110):

  • Matter concerned with financial matters like the imposition, abolition, remission, alteration or regulation of any tax.
  • Can be introduced Only in Lok
  • Introduced only on the recommendation of the President.
  • Cannot be amended or rejected by the Rajya Sabha should return the bill with or without recommendation Lok Sabha may accept or reject recommendation.
  • Rajya Sabha can keep it for a maximum period of 14 days.
  • Requires the certification of the
  • Rajya Sabha has limited power with respect to money bills so no chance of any
  • The President can reject or approve but cannot return it to parliament for

 

Question  7

Consider the following statements:

1. The Speaker of the Legislative Assembly shall vacate his/her office if he/she ceases to be a member of the assembly.

2. Whenever the Legislative Assembly is dissolved, the Speaker shall vacate his/her office immediately.

Which of the statements given above is/are correct?

(a) 1 only

(b) 2 only

(c) Both 1 and 2

(d) Neither 1 nor 2

Ans: a       

Sub-Theme: Speaker & Deputy Speaker of Assembly

  • Statement 1 is correct: The Speaker of the Legislative Assembly vacates his office in three cases: 1. If he ceases to be a member of the assembly; 2. Speaker Resigns by writing to the deputy speaker and vice versa and 3. Removed by the resolution passed by a majority of all the members of the assembly.
  • Statement 2 is incorrect: Article 179 provided that, whenever the Assembly is dissolved, the Speaker shall not vacate his office until immediately before the first meeting of the Assembly after the dissolution.

 

Question  8

Consider the following statements:

1. The Parliament of India can place a particular law in the Ninth Schedule of the Constitution of India.

2. The validity of a law placed in the Ninth Schedule cannot be examined by any court and no judgement can be made on it.

Which of the statements given above is/are correct?

(a) 1 only

(b) 2 only

(c) Both 1 and 2

(d) Neither 1 nor 2

Ans: a

Sub-Theme: Judicial Review

  • Statement 1 is correct: The Ninth Schedule and Article 31-B of the Indian Constitution was added by the 1st Amendment (1951) to protect the laws included in it from judicial scrutiny on the ground of violation of fundamental rights.
    • A law receives protection under Article 31-B (validation of certain Acts and Regulations) once it is passed and inserted into the Ninth Schedule and is thus exempt from judicial review.
  • Statement 2 is incorrect: The mandate of ninth schedule is to prevent judicial scrutiny but according to the Kesavananda Bharati Case, 1973, any violation with the ‘basic structure’ of the Constitution is open to challenge. Laws after 24 April 1973 are open for judicial review by courts.
    • IR Coelho vs State of Tamil Nadu, 2007: the SC ruled that all laws (including those in the Ninth Schedule) would be open to Judicial Review if they violated the basic structure of the constitution.

 

Question  9

Which of the following are regarded as the main features of the “Rule of Law”? 

1. Limitation of Powers

2. Equality before law

3. People’s responsibility to the Government

4. Liberty and civil rights

Select the correct answer using the code given below:

(a) 1 and 3 only

(b) 2 and 4 only

(c) 1, 2 and 4 only

(d) 1, 2, 3 and 4

Ans: c

Sub-Theme: Rule of Law

  • Statement 1 and 2 are correct: Rule of law: The concept of ‘equality before law’ as enshrined in Article 14 of Indian Constitution is taken from the concept of ‘Rule of Law’ propounded by A.V. Dicey.
  • Rule of law is a legal principle that states that a country should be ruled by the law rather than by the arbitrary actions of particular government officials.
  • The ‘Rule of Law’ has 3 fundamental principles:
    • Absence of arbitrary power
    • Equality before law
    • Primacy of individual rights
  • Statement 3 is incorrect: The government is responsible/accountable to the people and not the vice versa.

 

Question  10

Right to Privacy is protected as an intrinsic part of Right to Life and Personal Liberty. Which of the following in the Constitution of India correctly and appropriately imply the above statement?

(a) Article 14 and the provisions under the 42nd Amendment to the Constitution

(b) Article 17 and the Directive Principles of State Policy in Part IV

(c) Article 21 and the freedoms guaranteed in Part III

(d) Article 24 and the provisions under the 44th Amendment to the Constitution

Ans: c

Sub-Theme: Right to Privacy

Article 21 of the Indian Constitution deals with the protection of life and personal liberty. In the KS Puttaswamy Case (2017), the Supreme Court held that privacy is a natural right that inheres in all natural persons, and that the right may be restricted only by state action that passes each of the three tests:

  • Such state action must have a legislative
  • It must be pursuing a legitimate state purpose; and
  • It must be proportionate.

Additional Information:

  • Article 14 (Right to Equality): the state shall not deny to any person equality before the law or the equal protection of the laws within the territory of India.
  • Right to Equality = Equality before Law + Equal Protection of laws.
  • The concept of ‘equality before law’ as enshrined in Article 14 of Indian Constitution is taken from the concept of ‘Rule of Law’ propounded by V. Dicey.
  • Article 17 (Abolition of Untouchability): This Article forbids untouchability (not defined in Constitution or in the act) in any

 

2017

 

Question 1

Right to vote and to be elected in India is a

(a) Fundamental Right

(b) Natural Right

(c) Constitutional Right

(d) Legal Right

Ans: c

Sub-Theme: Universal Adult Franchise

Constitutional Right:

  • While framing the Indian Constitution the framers and makers of our Constitution took the decision to guarantee every adult Indian citizen to franchise their vote e., the Right to Vote.
  • In India, the Right to Vote is provided both by the Indian Constitution and the Representation of People’s Act, 1951.
  • Article 326: ensures that every individual above the age of 18 has the right to vote.
  • Section 62 of the Representation of Peoples Act (RoPA), 1951: the right to vote will be available to every person on the electoral roll of that constituency.
  • 61st Constitutional Amendment: lowered the voting age of elections to the Lok Sabha and to the Legislative Assemblies of States from 21 years to 18 years.

 

Question  2

Out of the following statements, choose the one that brings out the principle underlying the Cabinet form of Government:

(a) An arrangement for minimising criticism against the Government whose responsibilities are complex and hard to carry out to the satisfaction of all.

(b) A mechanism for speeding up the activities of the Government whose responsibilities are increasing day by day.

(c) A mechanism of parliamentary democracy for ensuring collective responsibility of the Government to the people.

(d) A device for strengthening the hands of the head of the Government whose hold over the people is in a state of decline.

Ans: c

Sub-Theme: Forms of Government

Parliamentary System:

  • The Parliamentary System is also referred to as Cabinet Government. It establishes a system of collective responsibility between the executive and the legislative.
  • Parliamentary system is part of the basic structure of the
  • The characteristics of India’s parliamentary system are:
    • Presence of nominal and real executives i.e., President and Prime minister respectively,
    • Majority party rule,
    • Collective responsibility (Swim or Sink together) of the executive to the legislature,
    • Membership of the ministers in the legislature,
    • Leadership of the prime minister or the chief minister,
    • Dissolution of the lower House (Lok Sabha or State Assembly).
  • It is through these powers that the Parliament enforces the responsibility of the executive to itself and to the people in the ultimate analysis.

 

Question  3

Which one of the following is not a feature to Indian federalism?

(a) There is an independent judiciary in India.

(b) Powers have been clearly divided between the Centre and the States.

(c) The federating units have been given unequal representation in the Rajya Sabha.

(d) It is the result of an agreement among the federating units.

Ans: d

Sub-Theme: Forms of Government

  • Option (a), (b) and (c) are correct: The Indian Constitution adopts the federal system of Government, but with more tilt towards a unitary system of government. So, every state government does not have power of its own. Thus, it is considered as a quasi-federal system.
  • The features of a federation are:
    • Two Government,
    • Division of powers,
    • Written Constitution,
    • Supremacy of Constitution,
    • Rigidity of Constitution,
    • Independent judiciary,
    • Bicameralism
  • Option (d) is incorrect: The phrase ‘Union of States’ has been preferred to ‘Federation of States’ for two reasons: One, unlike the American Federation (“Indestructible union of indestructible states”), the Indian Federation (“Indestructible union of destructible states”) is not the outcome of an agreement among the states; and two, the states are not permitted to secede the
  • Due to its invincibility, the federation is a Union. The country is a union that cannot be broken, although states can be split up and reorganised by changing their borders. State names and borders may be changed by the central government without their consent. B. R. Ambedkar described India as an “indestructible Union of destructible states” for this reason.

 

Question 4

The main advantage of the parliamentary form of governments is that:

(a) the executive and legislature work independently

(b) it provides continuity of policy and is more efficient.

(c) the executive remains responsible to the legislature.

(d) the head of the government cannot be changed without election.

Ans: c

Sub-Theme: Forms of Government

Parliamentary System:

  • The Parliamentary system is also known as the Cabinet Government/Responsible government/Westminster model of government. It provides for collective responsibility of the executive to the
  • The features of parliamentary government in India are:
    • Presence of nominal and real executives.
    • Majority party rule,
    • Collective responsibility of the executive to the legislature,
    • Membership of the ministers in the legislature,
    • Leadership of the prime minister or the chief minister,
    • Dissolution of the lower House (Lok Sabha or Assembly).

 

Question 5

Which one of the following objectives is not embodied in the Preamble to the Constitution of India?

(a) Liberty of thought

(b) Economic liberty

(c) Liberty of expression

(d) Liberty of belief

Ans: b

Sub-Theme: Key Words in the Preamble

Liberty as mentioned in the Preamble:

  • This ideal of ‘liberty’ in the Preamble to the Indian Constitution was borrowed from the French revolution.
  • It means the absence of restraints on the activities of individuals and at the same time, it provides opportunities for the development of individuals.
  • The preamble to the Indian Constitution assures to all citizens-
    • liberty of thought,
    • liberty of expression, liberty of belief, liberty of faith and worship Justice as mentioned in the Preamble:
  • The ideal of justice in the Indian Constitution embraces three distinct forms- social, economical and political:
    • Social Justice: the equal treatment of all citizens without any social discrimination.
    • Economic Justice: It calls for the non- discrimination between people on the basis of economic factors.
    • Political justice: It calls for equal access to political office, equal political rights, and equal voice in the government for all citizens.

 

Question 6

The mind of the makers of the Constitution of India is reflected in which of the following?

(a) The Preamble

(b) The Fundamental Rights

(c) The Directive Principles of State Policy

(d) The Fundamental Duties

Ans: a

Sub-Theme: Significance of the Preamble

Option (a) is correct: The Preamble:

  • Though India followed the practice of the American Constitution, the preamble to the Indian Constitution embodies the basic philosophy and the fundamental values on which our constitution is based.
  • It is not only the outcome of the grand and noble vision of the Constituent Assembly, but also the reflection of the dreams and aspirations of the founding fathers of the
  • In the Berubari Union case (1960), the Supreme Court opined that the Preamble is “a key to the minds of the makers of the ”

Option (b) is incorrect: Fundamental Rights:

  • The framers and makers of the Constitution derived inspiration of ‘fundamental rights’ from the Constitution of the USA (Bill of Rights) and also from the Motilal Nehru committee Report 1928. It was meant for promoting the ideal of political democracy.

Option (c) is incorrect: Directive Principles of State Policy (DPSP):

  • It resembles the ‘Instrument of Instructions’ enumerated in GoI Act 1935. They are intended to fill in the vacuum in Part III by providing for social and economic

Option (d) is incorrect: Fundamental Duties:

  • Inspired from the erstwhile USSR, the Fundamental Duties were added to the Constitution by 42nd Constitutional Amendment Act, 1976.

 

Question 7

Which of the following are envisaged by the Right against Exploitation in the Constitution of India?

1. Prohibition of traffic in human beings and forced labour

2. Abolition of untouchability

3. Protection of the interests of minorities

4. Prohibition of employment of children in factories and mines

Select the correct answer using the code given below:

(a) 1, 2 and 4 only

(b) 2, 3 and 4 only

(c) 1 and 4 only

(d) 1, 2, 3 and 4

Ans: c

Sub-Theme: Right against Exploitation

Statement 1 is correct: As under article 23 (Right Against Exploitation) of the constitution, the expression ‘traffic in human beings’ includes: Selling and buying of men, women and children like goods, Immoral traffic in women and children, including prostitution, Devadasis, Slavery.

Statement 2 is incorrect: Abolition of Untouchability is provided in the constitution of India, but it comes under Right to Equality, under article 17.

Statement 3 is incorrect: The Cultural and Educational Rights (under articles 29 and 30) cover protection of interests of minorities; they don’t come under Rights Against Exploitation.

Statement 4 is correct: Article 24 (Right Against Exploitation) ensures prohibition of employment of children in factories etc. The Commissions for Protection of Child Rights Act, 2005 was enacted to provide for the establishment of a National Commission and State Commissions for Protection of Child Rights and Children’s Courts for providing speedy trial of offences against children or of violation of child rights.

NOTE: Here option 4 is present in all given options. Observing such things will save your time and lessen the chances of error.

 

Question 8

In the context of India, which one of the following is the correct relationship between Rights and Duties?

(a) Rights are correlative with duties.

(b) Rights are personal and hence independent of society and Duties.

(c) Rights, not Duties, are important for the advancement of the personality of the citizen.

(d) Duties, not Rights, are important for the stability of the State.

Ans: a

Sub-Theme: Correlation between Rights and Duties

  • In the context of India, Rights are correlative with duties, considering the Indian political set up.
  • In a constitutional democracy (such as India), the State provides rights and lays down duties for the citizenry, so that society can peacefully However, sometimes the State imposes such duties on its citizens that infringes the rights of the marginalised sections of society.
  • Rights and duties have an organic relationship. If others do not carry out their duties, one cannot enjoy a right.
  • For Example: One’s right to life implies that others should give protection and security to the former’s Similarly, One cannot enjoy their Right to a clean environment if others do not perform their duty of protecting and conserving the environment.

 

Question 9

One of the implications of equality in society is the absence of:

(a) Privileges

(b) Restraints

(c) Competition

(d) Ideology

Ans: a

Sub-Theme: Right to Equality

Equality in the society can be maintained by ending the formal system of inequality and ensuring absence of privileges. The lack of any special privileges for any individual is a concept related to Equality Before Law (which is an American Concept and denotes positive notion). This concept entails equal subjection of all persons to the ordinary law of the land administered by ordinary law courts, and no one is above the law, regardless of their status (rich or poor, high or low, official or not).

Additional Information:

  • Absence of Restraint does not necessarily imply equality in society as restraining the actions of the society curtails the liberty of the individual.
  • Absence of Competition in society may result in monopoly, which would not imply equality, rather inequality in the society.
  • Absence of ideology does not imply equality in society. Ideology is a set of principles or beliefs of a group/institution in the society.

 

Question 10

Which one of the following statements is correct?

(a) Rights are claims of the State against the citizens.

(b) Rights are privileges which are incorporated in the Constitution of a State.

(c) Rights are claims of the citizens against the State.

(d) Rights are privileges of a few citizens against the many.

Ans: c

Sub-Theme: Rights of Citizen

Option (a) is incorrect: Rights are claimed by the citizens against the state and not the vice versa.

Option (b) is incorrect: Rights are not privileges either, they are one of the quintessential requirements for the formation of a democratic state. Thus, the constitution has no concept of special privileges which will create inequality in the society.

Option (c) is correct: The philosophy of rights is grounded in preventing the exploitation of the citizen by the state. It inherently belongs to the citizens. So, essentially it is the claim of an individual against the state.

Option (d) is incorrect: As per Article 18, except for military and academic distinctions, no other citizens have titular privileges. Hence, Rights are not the privileges of a few citizens as they are equally available to all.

 

Question 11

Which principle among the following was added to the Directive Principles of State Policy by the 42nd Amendment to the Constitution?

(a) Equal pay for equal work for both men and women

(b) Participation of workers in the management of industries

(c) Right to work, education and public assistance

(d) Securing living wage and human conditions of work to workers

Ans: b

Sub-Theme: Amendments to Constitution

  • 42nd Amendment Act of 1976 added four new Directive Principles to the original list, and they are –
    • Article 39: To secure opportunities for healthy development of children
    • Article 39 A: To promote equal justice and to provide free legal aid to the poor.
    • Article 43 A: To take steps to secure the participation of workers in the management of industries.
    • Article 48 A: To protect and improve the environment and to safeguard forests and wildlife.

 

Question 12

Consider the following statements:

With reference to the Constitution of India, the Directive Principles of State Policy constitute limitations upon 

1. legislative

2. executive

Which of the above statements is/are correct?

(a) 1 only

(b) 2 only

(c) Both 1 and 2

(d) Neither 1 nor 2

Ans: d

Sub-Theme: Features of the Directive Principles

  • DPSP: The directive principles are in the nature of instruments of instruction to the government of the day to achieve certain ends by their actions. Directive Principles of State Policy does not impose any limitations on any organ of the State. It is a guideline that the government of the day can use for the welfare of the socially and economically backward classes while framing any rules.
  • Fundamental Rights: It is the fundamental rights that constitute limitations upon state action (whether legislative or executive).

 

Question 13

Which of the following statements is/are true of the Fundamental Duties of an Indian citizen?

1. A legislative process has been provided to enforce these duties.

2. They are correlative to legal duties.

Select the correct answer using the code given below:

(a) 1 only

(b) 2 only

(c) Both 1 and 2

(d) Neither 1 nor 2

Ans: d

Sub-Theme: Basics of Fundamental Duties

Statement 1 is incorrect: Fundamental Duties are enumerated in Part IV-A and consists of single Article 51A with 11 sub-articles. These duties should guide the legislative and executive actions of elected or non-elected institutions and organisations of the citizens including the municipal bodies. But no legislative process has been provided to enforce these duties. These duties are thus non-enforceable and non- justiciable in nature.

Statement 2 is incorrect: The fundamental Duties are not correlative of the legal duties. Parliament may provide legislation for the imposition of penalty or punishment for non- observance or infringement of duties.

NOTE: Fundamental Duties are confined to citizens only & not to foreigners.

 

Question 14

Which of the following are not necessarily the consequences of the proclamation of the President’s rule in a State?

1. Dissolution of the State Legislative Assembly

2. Removal of the Council of Ministers in the State

3. Dissolution of the local bodies

Select the correct answer using the code given below:

(a) 1 and 2 only

(b) 1 and 3 only

(c) 2 and 3 only

(d) 1, 2 and 3

Ans: b

Sub-Theme: State Emergency/President’s Rule

Statement 1 is correct: The proclamation of the President’s Rule is also known as ‘State Emergency’ or ‘Constitutional Emergency’. As per article 356 of the Constitution of India, the president has the authority to declare if he is satisfied that the state government cannot function in accordance with the Constitution’s provisions. (The President can act on the report of the governor or otherwise). But the ground for Dissolution of the State Legislative Assembly is not necessarily the consequence of proclamation.

Statement 2 is incorrect: When the President’s Rule is imposed in a state, the President dismisses the state council of ministers headed by the chief minister.

Statement 3 is correct: There is no mention of Dissolution of the local bodies during the proclamation of President’s Rule.

 

Question 15

Democracy’s superior virtue lies in the fact that it calls into activity

(a) the intelligence and character of ordinary men and women.

(b) the methods for strengthening executive leadership.

(c) a superior individual with dynamism and vision.

(d) a band of dedicated party workers.

Ans: a

Sub-Theme: Basic Idea of Democracy (Conceptual)

The basic idea of democracy is the desire, approval and participation of people. A democratic government is created, and its operations are decided by the will of the people. It is the decision of people that creates a democratic government and decides about its functioning. Therefore, democracy requires voters’ decision making – hence, intelligence and character are called in.

 

Question 16

For election to the Lok Sabha, a nomination paper can be filed by:

(a) anyone residing in India.

(b) a resident of the constituency from which the election is to be contested.

(c) any citizen of India whose name appears in the electoral roll of a constituency.

(d) any citizen of India.

Ans: c

Sub-Theme: Qualification of Lok Sabha

There are some qualification criteria mentioned in the Indian Constitution for filing the nomination for Lok Sabha election and those are –

  1. The candidate must be a citizen of
  2. The age of the candidate should not less than 25 years of age
  3. The candidate must be registered as an elector for a parliamentary constituency.
NOTE: This a straightforward question, a proper reading of M. Laxmikanth could help in giving the answer.

 

Question 17

Consider the following statements:

1. In the election for Lok Sabha or State Assembly, the winning candidate must get at least 50 percent of the votes polled, to be declared elected.

2. According to the provisions laid down in the Constitution of India, in Lok Sabha, the Speaker’s post goes to the majority party and the Deputy Speaker’s to the Opposition.

Which of the statements given above is/are correct?

(a) 1 only

(b) 2 only

(c) Both 1 and 2

(d) Neither 1 nor 2

Ans: d

Sub-Theme: People’s Representative/Speaker and Deputy Speaker

Statement 1 is incorrect: The Indian constitution adopts the first-past-the-post (FPTP) system of elections where the majority of votes is sufficient to get elected, even if it is less than 50% of the total votes polled.

Statement 2 is incorrect: Speaker and Deputy speaker are elected by the house. This is a convention (not constitutional provision) that usually the Speaker’s post goes to the majority party and the Deputy Speaker’s to the Opposition.

 

Question 18

The Parliament of India exercises control over the functions of the Council of Ministers through

1. Adjournment motion

2. Questions hour

3. Supplementary questions

Select the correct answer using the code given below:

(a) 1 only

(b) 2 and 3 only

(c) 1 and 3 only

(d) 1, 2 and 3

Ans: d

Sub-Theme: Devices of Parliamentary Proceedings

Devices Of Parliamentary Proceeding

  • Adjournment Motion: It is introduced to draw attention of the House to a definite matter of urgent public importance which is definite, factual, urgent, matter of recent
  • Question Hour: In the First Hour of parliamentary sitting usually members ask questions and ministers give answers but sometimes questions can be asked to private members too.
  • Three types of Question are asked namely–
    • Starred Question: Require oral answer and Supplementary question can be
    • Unstarred Question: Require written answer and Supplementary question cannot be asked.
    • Short Notice Question: Answered orally and asked on short notice of less than 10 days.

Thus, Adjournment Motion, Question Hour and Supplementary Question, all constitutes devices of the Parliament of India through which it exercises control over the functions of the Council of Ministers.

 

Question 19

With reference to the Parliament of India, consider the following statements:

1. A private member’s bill is a bill presented by a Member of Parliament who is not elected but only nominated by the President of India.

2. Recently, a private member’s bill has been passed in the Parliament of India for the first time in its history.

Which of the statements given above is/are correct?

(a) 1 only

(b) 2 only

(c) Both 1 and 2

(d) Neither 1 nor 2

Ans: d

Sub-Theme: Legislative Procedure of Parliament

Statement 1 is incorrect: A Private Member bill can be introduced by any member of the Parliament other than a minister. Whereas, a bill introduced by a Minister in the Parliament is called a public bill. A Private member bill generally reflects the stand of the opposition party on public matters. There is less likelihood that the Parliament will adopt it. Its rejection by the House has no implication on the parliamentary confidence in the government or its resignation.

Statement 2 is incorrect: As of now around 14 private member’s bills have been passed since independence in Indian Parliament.

 

Question 20

In India, Judicial Review implies:

(a) the power of the Judiciary to pronounce upon the constitutionality of laws and executive orders.

(b) the power of the Judiciary to question the wisdom of the laws enacted by the Legislatures.

(c) the power of the Judiciary to review all the legislative enactments before they are assented to by the President.

(d) the power of the Judiciary to review its own judgements given earlier in similar or different cases

Ans: a

Sub-Theme: Judicial Review

Judicial Review:

  • In general terms, judicial review refers to the power and ability of High Courts and Supreme Court to review laws or judgments to ensure that they do not violate constitutional or legal provisions.
    • Article 13: states that, Courts can review government orders, legislations, byelaw, rule, regulation, notification or any law in force in Indian Territory.
    • Article 32/Article 226: allows a person to move to SC/HC in case of violation of fundamental rights. Thus, SC & HC can issue writs as constitutional remedies against the wrongdoer.
    • Article 137: empowers SC to review judgments or orders made by the Supreme Court itself. The review will be done by a larger bench of SC.
    • Article 142: allows the Supreme Court to pass such a decree or make such order as is necessary for doing complete justice in any cause or matter pending before it.
  • Primarily, Article 13, 32 and 226 forms the core of judicial review, of which Article 13 and 32 forms part of Fundamental Right.
  • Further judicial review has also been considered as part of basic structure of the Indian Constitution and hence cannot be amended.
  • Thus, the Courts are empowered to declare a statute ultra vires the constitution and to nullify an executive action as unconstitutional if they violate constitutional provisions or alter fundamental rights.
  • These powers of judicial review are given not to make the judiciary superior, but to ensure a system of checks and balances between the legislature and the executive on one hand, and the judiciary on the

 

Question 21

Local self-government can be best explained as an exercise in

(a) Federalism

(b) Democratic decentralisation

(c) Administrative delegation

(d) Direct democracy

Ans: b      

Sub-Theme: Objective of Local Governance

Option (b) is correct: The main idea behind the Panchayati Raj Institutions (PRIs) or Local Self-Government is to promote grass root democracy and development. The main aim is to evolve a system of democratic decentralisation and people’s participation with a view to ensure rapid socio-economic progress and provide adequate justice.

NOTE: Phrase to emphasis here is “Best”. Democratic decentralisation is the best description of Local self-government. Balwant Rai Mehta committee (on Community Development Program) in 1957 recommended the establishment of a scheme of Democratic decentralisation with 3-tiered structure of PRI.

 

Question 22

Consider the following statements:

1. The Election Commission of India is a five- member body.

2. The Union Ministry of Home Affairs decides the election schedule for the conduct of both general elections and bye-elections.

3. Election Commission resolves the disputes relating to splits/mergers of recognized political

Which of the statements given above is/are correct?

(a) 1 and 2 only

(b) 2 only

(c) 2 and 3 only

(d) 3 only

Ans: d

Sub-Theme: Election Commission of India

Statement 1 is Incorrect: Strength of ECI is not specified in the Constitution, it is at the discretion of the President. Currently, there are 3 Members.

Statement 2 is Incorrect: The most important function of the commission is to decide the election schedules for the conduct of periodic and timely elections, whether general or bye- elections.

Statement 3 is Correct: It grants recognition to political parties & allot election symbols to them along with settling disputes related to it.

 

2016

 

Question 1

The Parliament of India acquires the power to legislate on any item in the State List in the national interest if a resolution to that effect is passed by the:

(a) Lok Sabha by a simple majority of its total membership.

(b) Lok Sabha by a majority of not less than two-thirds of its total membership

(c) Rajya Sabha by a simple majority of its total membership

(d) Rajya Sabha by a majority of not less than two- thirds of its members present and voting

Ans: d

Sub-Theme: Legislative Power of Parliament

Legislative Power

  • In accordance with the Constitution the Parliament has the authority to enact legislation on all matters listed in the Union List and Concurrent List.
  • If the Rajya Sabha passes a resolution to that effect with not less than two-thirds of the members present and voting, the Union Parliament may legislate on any item on the State List in the interest of the country.
  • As per Article 250, it can also formulate laws over the State List when an Emergency is declared in the country, or President’s Rule is declared in any State.
  • Article 253 says that it can also formulate laws on any matter pertaining to the State List if it is deemed necessary for the implementation of international treaties or agreements concluded with foreign
  • Apart from these legislative powers the Parliament also enjoys the Executive Power, Financial Powers, Constituent Powers, Judicial Powers, Electoral Powers, and other

 

Question 2

Which of the following statements is/are correct?

1. A Bill pending in the Lok Sabha lapses on its prorogation.

2. A Bill pending in the Rajya Sabha, which has not been passed by the Lok Sabha, shall not lapse on dissolution of the Lok Sabha.

Select the correct answer using the code given below.

(a) 1 only

(b) 2 only

(c) Both 1 and 2

(d) Neither 1 nor 2

Ans: b

Sub-Theme: Legislative Procedure of Parliament

  • Statement 1 is incorrect: No bill lapses on prorogation. Prorogation does not affect thebills or any other business pending before the House.
  • Statement 2 is correct: A Bill pending in the Rajya Sabha, which has not been passed by the Lok Sabha, shall not lapse on dissolution of the Lok Sabha.
  • Additional Information:
    • Bill lapses:
      • Bills pending in the Lok Sabha (whether originating in the Lok Sabha or transmitted to it by the Rajya Sabha)
      • Bills passed by the Lok Sabha but pending in the Rajya (Note – A bill originating in Lok Sabha lapses)
    • Bills does not lapse:
      • Bill not passed by the two Houses due to disagreement and if the president has notified the holding of a joint sitting before the dissolution of Lok Sabha.
      • Bill pending in the Rajya Sabha but not passed by the Lok Sabha.
      • Bill passed by both Houses but pending assent of the president.
      • Bill passed by both Houses but returned by the president for reconsideration of

 

Question 3

Consider the following statements:

1. The Chief Secretary in a State is appointed by the Governor of that State.

2. The Chief Secretary in a State has a fixed tenure.

Which of the statements given above is/are correct?

(a) 1 only

(b) 2 only

(c) Both 1 and 2

(d) Neither 1 nor 2

Ans: d

Sub-Theme: State Executives

Statement 1 is incorrect: The Chief Secretary is ‘chosen’ by the Chief Minister. As the appointment of Chief Secretary is an executive action of the Chief Minister, it is taken in the name of the Governor of the State.

Statement 2 is incorrect: Though the Administrative Reforms Commission, in its report on State Administration in 1969, had recommended that a Chief Secretary should have a minimum tenure of three to four years, there is no fixed tenure for the post of Chief Secretary of a State.

 

Question 4

Consider the following statements:

1. The minimum age prescribed for any person to be a member of Panchayat is 25 years.

2. A Panchayat reconstituted after premature dissolution continues only for the remainder period.

Which of the statements given above is/are correct?

(a) 1 only

(b) 2 only

(c) Both 1 and 2

(d) Neither 1 nor 2

Ans: b

Sub-Theme: Basics of Panchayati Raj System

Statement 1 is incorrect: Part IX of the Constitution deals with the Panchayati Raj System in India. The Panchayati Raj was constitutionalized in India through the 73rd Constitutional Amendment Act of 1992. As per the act, no person shall be disqualified on the ground that he is less than 25 years of age if he has attained the age of 21 years. Therefore, the minimum age is 21.

Statement 2 is correct: Panchayat reconstituted after premature dissolution does not enjoy the full period of five years but remains in office only for the remaining period.

 

Question 5

With reference to the ‘Gram Nyayalaya Act’, which of the following statements is/are correct? 

1. As per the Act, Gram Nyayalayas can hear only civil cases and not criminal cases.

2.The Act allows local social activists as mediators/ re-conciliators.

Select the correct answer using the code given below.

(a) 1 only

(b) 2 only

(c) Both 1 and 2

(d) Neither 1 nor 2

Ans: b     

Sub-Theme: Judicial Power of Panchayat

Statement 1 is incorrect: The establishment, jurisdiction, and process in civil and criminal proceedings are all outlined in the Gram Nyayalayas Act. As per the Gram Nyayalaya Act, the Gram Nyayalayas can hear both the civil and criminal cases.

Statement 2 is correct: The Court shall try to settle disputes via conciliation between the parties and the court can make use of the conciliators (social workers) to be appointed for this purpose.

 

2015

 

Question 1

The provisions in the Fifth Schedule and Sixth Schedule in the Constitution of India are made in order to:

(a) protect the interests of Scheduled Tribes

(b) determine the boundaries between States

(c) determine the powers, authority and responsibilities of Panchayats

(d) protect the interests of all the border States

Ans: a

Sub-Theme: Articles and Schedules of Indian Constitution

The fifth and the sixth schedule of the Constitution of India deals with the provisions related to the protection of the interests of the Scheduled Tribes.

  • Fifth Schedule: Deals with the provisions relating to the administration and control of scheduled areas and scheduled tribes in any state except 4 states of Assam, Meghalaya, Tripura & Mizoram (AMTM).
  • Article 244 (1) of the Constitution defines Scheduled Areas as the areas defined so by the President of India and are mentioned in the 5th schedule of the Constitution.
  • Sixth Schedule: It deals with the provisions as to the Administration of Tribal Areas in 4 NE states of Assam, Meghalaya, Tripura & Mizoram (Not Manipur).

 

Question 2

Consider the following statements regarding the Directive Principles of State Policy:

1. The principles spell out of the socio-economic democracy in the country.

2. The provisions contained in these Principles are not enforceable by any court.

Which of the statements given above is/are correct?

(a) 1 only

(b) 2 only

(c) Both 1 and 2

(d) Neither 1 nor 2

Ans: c

Sub-Theme: Features of the Directive Principles

  • Statement 1 is correct: The Directive Principles deals with a very comprehensive economic, social and political programme for a modern democratic State. They aim at realising the high ideals of justice, liberty, equality and fraternity as outlined in the Preamble to the Constitution.
    • Article 38: Deals with the promotion of the welfare of the people by securing and protecting a social order by ensuring social, economic and political justice and by minimising inequalities in income, status, facilities and opportunities.
    • R. Ambedkar opined that the Directives have great value because they lay down that the goal of Indian polity is ‘economic democracy’ as distinguished from ‘political democracy’.
  • Statement 2 is correct: According to Article 37 Directive Principles are not enforceable by any court, but the principles therein laid down are nevertheless fundamental in the governance of the country.

 

Question 3

The ideal of ‘Welfare State’ in the Indian Constitution is enshrined in its

(a) Preamble

(b) Directive Principles of State Policy

(c) Fundamental Rights

(d) Seventh Schedule

Ans: b

Sub-Theme: Features of the Directive Principles

    • The Directive Principles of State Policy (DPSP) are mentioned under Articles 36 to 51 of the Indian Constitution.
    • The DPSPs have been borrowed from the Constitution of Ireland. It constitutes a comprehensive socio economic and political programme for a modern democratic State. DPSP embodies the concept of a ‘welfare state’ and not that of a ‘police state’.
    • Article 38 (1): The State shall strive to promote the welfare of the people by securing and protecting as effectively as it may a social order in which justice, social, economic and political, shall inform all the institutions of the national life.

Additional Information:

  • Fundamental Rights: It is the basic human rights enshrined in Article 12 -35 of Part III of Indian They are enforceable by the courts, subject to certain conditions.
  • Preamble: It is the ideals and philosophy of the The Preamble gives an idea about the following:
    •  Source of the constitution
    • Nature of Indian state
    • Statement of its objectives
    • Date of its adoption
  • Seventh Schedule: The seventh schedule under Article 246 in Part XI of the Constitution deals with the division of powers between the union and the states. It contains three lists- Union List, State List and Concurrent List.

 

Question 4

“To uphold and protect the Sovereignty, Unity and Integrity of India” is a provision made in the:

(a) Preamble of the Constitution

(b) Directive Principles of State Policy

(c) Fundamental Rights

(d) Fundamental Duties

Ans: d

Sub-Theme: List of Fundamental Duties

  • Article 51A of part IV-A of the Indian Constitution deals with the Fundamental Duties. These duties are the moral and civic obligations that all citizens of a nation
  • In 1976, the Fundamental Duties were first recommended by the Swaran Singh Committee, its need was felt during the internal emergency (1975-77) and were added to the Constitution by 42nd Constitutional Amendment Act, 1976.
  • One of the Fundamental Duties is to “To protect the sovereignty, unity and integrity of India” among the 11 Fundamental

 

Question 5

There is a Parliamentary System of Government in India because the

(a) Lok Sabha is elected directly by the people

(b) Parliament can amend the Constitution

(c) Rajya Sabha cannot be dissolved

(d) Council of Ministers is responsible to the Lok Sabha

Ans: d

Sub-Theme: Parliamentary System of Government

India has a parliamentary system of government. The Union Parliament is the supreme legislative body in the country and in a Parliamentary system, the executive is responsible for Parliament. In India, this system is applicable to both Union and State level. The Prime Minister and his Council of Ministers (CoM) are responsible to the Lok Sabha. This is called Collective Responsibility, which is considered as the bedrock of Parliamentary Government. Article 75 of the Constitution of India states that CoM is collectively responsible to the Lok Sabha. The Lok Sabha can remove CoM by passing no confidence motion.

 

Question 6

Consider the following statements:

1. The Executive Power of the Union of India is vested in the Prime Minister.

2.The Prime Minister is the ex Officio Chairman of the Civil Services Board.

Which of the statements given above is/are correct?

(a) 1 only

(b) 2 only

(c) Both 1 and 2

(d) Neither 1 nor 2

Ans: d

Sub-Theme: Powers of President/Executives

Statement 1 is incorrect: As per the Article 53(1), the executive power of the Union shall be vested in the President and shall be exercised by him either directly or through officers subordinate to him in accordance with this Constitution.

Statement 2 is incorrect: The Cabinet Secretary is the top-most executive official and senior-most civil servant of the Government of India and he/she is the ex-officio head of the Civil Services Board.

 

Question 7

Consider the following statements:

1. The Rajya Sabha has no power either to reject or to amend a Money Bill.

2. The Rajya Sabha cannot vote on the Demands for grants.

3. The Rajya Sabha cannot discuss the Annual Financial Statement.

Which of the statements given above is/are correct?

(a) 1 only

(b) 1 and 2 only

(c) 2 and 3 only

(d) 1, 2 and 3

Ans: b

Sub-Theme: Annual Financial Statement / Money Bill

  • Statement 1 is correct: The Rajya Sabha can keep the Money Bill for a maximum period of 14 days. The house cannot amend or reject the Money Bill, it should return the bill with or without recommendation to Lok Sabha, after that Lok Sabha may accept or reject recommendation.
  • Statement 2 is correct: The Rajya Sabha has no power to vote on the demand for grants; it is the exclusive privilege of the Lok Sabha.
  • Statement 3 is incorrect: The Rajya Sabha enjoys equal status with the Lok Sabha on the matter of discussion of Annual Financial Statement.
NOTE: In previous years, many questions have been asked on Money bill, Annual Financial Statement and role of Rajya Sabha and President in this regard. These are highly important topics.

 

Question 8

When a bill is referred to a joint sitting of both the Houses of the Parliament, it has to be passed by

(a) a simple majority of members present and voting

(b) three-fourths majority of members present and voting

(c) two-thirds majority of the Houses

(d) absolute majority of the Houses

Ans: a

Sub-Theme: Joint Session of Parliament

Joint sitting (Article 108) of two house:

  • It is extraordinary machinery provided by the Constitution to resolve a deadlock between the two Houses over the passage of a bill –
    • If the bill is rejected by the other
    • If the Houses have finally disagreed as to the amendments to be made in the bill; or
    • If more than six months have elapsed from the date of the receipt of the bill by the other House without the bill being passed by it.
  • Summon: The president can summon joint sitting for the purpose of deliberating and voting on the bill.

 

Question 9

Who/Which of the following is the custodian of the Constitution of India?

(a) The President of India

(b) The Prime Minister of India

(c) The Lok Sabha Secretariat

(d) The Supreme Court of India

Ans: d

Sub-Theme: Interpretation of Constitution The Supreme Court as the Custodian of the Indian Constitution:

  • Indian Judiciary is an integrated, powerful and independent judiciary.
  • The Legislature and Executive works on making laws and implementation of the laws, but the Judiciary works on safeguarding the public interest by checking into overreach, misuse and error of law.
  • Therefore, the authority to amend the constitution lies with the Parliament, but the SC has the authority to examine the validity of those amendments.
  • SC has the power of Judicial
  • According to Article 32, the SC is the guardian of Fundamental Rights.
  • The SC also an interpreter and Guardian of the Constitution.

 

Question 10

Consider the following statements:

1. The Legislative Council of a State in India can be larger in size than half of the Legislative Assembly of that particular State.

2. The Governor of a State nominates the Chairman of the Legislative Council of that particular State.

Which of the statements given above is/are correct?

(a) 1 only

(b) 2 only

(c) Both 1 and 2

(d) Neither 1 nor 2

Ans: d

Sub-Theme: Composition of Legislative Assembly/Legislative Council

Statement 1 is incorrect: As per Article 171, the maximum strength of the Legislative Council of the state cannot be more than one third of total strength of assembly. However, the size of the Legislative Council cannot be less than 40 members. The constitution has fixed the maximum and minimum limits but actual strength is fixed by Parliament.

Statement 2 is incorrect: The members of the Legislative Council are indirectly elected. The Chairman of the Legislative Council is elected by the members from amongst itself.

 

Question 11

The fundamental object of the Panchayati Raj system is to ensure which among the following? 

1. People’s participation in development

2. Political accountability

3. Democratic decentralisation

4. Financial mobilisation

Select the correct answer using the code given below.

(a) 1, 2 and 3 only

(b) 2 and 4 only

(c) 1 and 3 only

(d) 1, 2, 3 and 4

Ans: c

Sub-Theme: Objective of Local Governance

  • Statements 1 and 3 are correct: The main idea behind the Panchayati Raj Institutions (PRIs) is to promote grass root democracy and development. The main aim is to evolve a system of democratic decentralisation and people’s participation with a view to ensure rapid socio-economic progress and provide adequate justice.
  • Statement 2 is incorrect: Political accountability can be achieved in any system with democratic rule.
  • Statement 4 is incorrect: Financial mobilisation was never the basis for the establishment of PRI.
NOTE: In this question, the catch word is “Fundamental object”. We must stress on this phrase and apply this litmus test to each given sentence. Political accountability is never a Fundamental objective of the Panchayati Raj system. Moreover, People’s participation in development and Democratic decentralisation are correlative to each other. Financial mobilisation is meant for financial autonomy of PRI, but not a fundamental objective.

 

2014

 

Question 1

Which one of the following Schedules of the Constitution of India contains provisions regarding anti-defection?

(a) Second Schedule

(b) Fifth Schedule

(c) Eighth Schedule

(d) Tenth Schedule

Ans: d

Sub-Theme: Articles and Schedules of Indian Constitution

Tenth Schedule:

  • It deals  with  the  provisions  as  to disqualification on grounds of defection.
  • In 1985, the 52nd Amendment Act added the Tenth Schedule to the Constitution.
  • On the basis of a petition by any other member of the House, it lays out the procedure by which a legislator may be removed from office on the grounds of defection by the Presiding officer of a legislature.
  • The Chairman or Speaker of that House is tasked with making the final determination regarding disqualification due to defection.
  • Both the Parliament and state legislatures are subject to the statute.
  • The Presiding Officer’s decision is not subject to judicial review, according to the original law.
  • This condition was struck down by the Supreme Court in Kihoto Hollohan judgement (1992), thereby allowing appeals against the Presiding Officer’s decision in the High Court and Supreme Court. Judicial review is a basic feature of the constitution.

 

Question 2

In the Constitution of India, promotion of international peace and security is included in the

(a) Preamble to the constitution

(b) Directive Principles of State Policy

(c) Fundamental Duties

(d) Ninth Schedule

Ans: b

Sub-Theme: Liberal-Intellectual Principles

The Directive Principles of State Policy (DPSP) are mentioned under Articles 36 to 51 of the Indian Constitution. The DPSPs have been borrowed from the Constitution of Ireland. Article 51 is clear in dealing with the international peace and security and for achieving the same it directs the State to focus on:

  • Maintaining just and honourable relations with the nations.
  • Foster respect for international law and treaty obligations.
  • Encourage settlement  of international disputes by arbitration.

 

Question 3

Consider the following statements – A Constitutional Government is one which:

1.  Places effective restrictions on individual liberty in the interest of State Authority.

2. Places effective restrictions on the Authority of the State in the interest of individual liberty.

Which of the statements given above is/are correct?

(a) 1 only

(b) 2 only

(c) Both 1 and 2

(d) Neither 1 nor 2

Ans: b

Sub-Theme: Constitutional Government

The Constitutional Government is defined by the existence of a constitution. The idea behind constitutionalism is to place restrictions on the authority of the state so that it does not become autocratic. Thus, the Constitutional Government places restrictions on the Authority of the State to protect the interest of individual liberty (Example: Fundamental Rights).

 

Question 4

Consider the following statements:

1. The President shall make rules for the more convenient transaction of the business of the Government of India, and for the allocation among Ministers of the said business.

2. All executive actions of the Government of India shall be expressed to be taken in the name of the Prime Minister.

Which of the statements given above is/are correct?

(a) 1 only

(b) 2 only

(c) Both 1 and 2

(d) Neither 1 nor 2

Ans: a

Sub-Theme: Powers of President

Statement 1 is correct: India follows a Parliamentary type of Government in which the President is the constitutional head of the state. He is the Executive head of India. Article 77(3) of the Indian Constitution says that the President shall make rules for the more convenient transaction of the business of the Government of India, and for the allocation among Ministers of the said business.

Statement 2 is incorrect: Article 77(1) of the Indian Constitution lays down all executive action of the Government of India shall be expressed to be taken in the name of the President.

 

Question 5

Consider the following statements regarding a No- Confidence Motion in India:

1. There is no mention of a No-Confidence Motion in the Constitution of India.

2. A motion of No-Confidence can be introduced in the Lok Sabha only.

Which of the statements given above is/are correct?

(a) 1 only

(b) 2 only

(c) Both 1 and 2

(d) Neither 1 nor 2

Ans: c

Sub-Theme: No-Confidence Motion

Statement 1 is correct: The Constitution of India does not mention about either a Confidence or a No Confidence Motion. A motion of “No Confidence Motion” against the Government can be introduced only in the Lok Sabha under rule 198. If the motion is accepted, then the party in power has to prove its majority in the House. The member need not give a reason for moving the no-confidence motion.

Statement 2 is correct: The party can remain in power when it shows its strength through a floor test which is primarily taken to know whether the executive enjoys the confidence of the legislature. If any member of the House feels that the government in power does not have a majority then he/she can move a no- confidence motion. A government can function only when it has majority support in the Lok Sabha.

Additional Information:

No confidence Motion Censure Motion
•   Art. 75: Council of ministers shall be collectively responsible to the Lok Sabha. This principle is the bedrock of parliamentary democracy.

•   Not mentioned in the Constitution, is moved under

Rule 198 of rules of procedure and can be moved only in Lok Sabha.

•   Ministry stays in office till they enjoy confidence of the majority of the members of the Lok Sabha

•   Needs support of 50 members, no need to state the reasons for its adoption.

Moved only against the entire CoM (Not against individual/

group of ministers) and if passed, the CoM must

resign from office.

•   Moved to seek the disapproval of certain policies of the government.

•   Need to state the

reasons for its adoption.

•   Can be moved against an individual minister or

a group of ministers or the entire council of ministers.

If it is passed in the Lok Sabha, the CoM need not resign from the office but the government to seek the confidence

of the house immediately.

 

Question 6

Which one of the following is the largest Committee of the Parliament? 

(a) The Committee on Public Accounts

(b) The committee on Estimates

(c) The Committee on Public Undertakings

(d) The committee on Petitions

Ans: b

Sub-Theme: Parliamentary Committees

The Estimates Committee was set up in 1950, on the recommendation of John Mathai. It consists of 30 members, all from the Lok Sabha and it is the largest committee. The minister cannot be the members of the estimates committee. Its function is to examine the budget and also suggest economies of public expenditure. It also suggests an alternative policy of bringing policy of economies.

Additional Information: Parliamentary Committee

  • The Constitution of India mentions these committees at different places, but without making any specific provisions regarding their composition, tenure, functions, etc. as it has been dealt in detail in the rules of two
  • Public Accounts Committee
  • Origin: set up first in 1921 under Govt. of India Act 1919
  • Composition: 22 (15 Lok Sabha + 7 Rajya Sabha) Elected for one year – proportional representation by single transferable vote. The Chairman – from Opposition Minister cannot be a member.
  • Function: To examine CAG Audit report and discover the irregularities

Committee on Public Undertakings

  • Origin: set up on the recommendations of Krishna Menon Committee 1964
  • Composition: 22 (15 Lok Sabha + 7 Rajya Sabha) Elected for one year – proportional representation by single transferable vote. The Chairman – the opposition minister cannot be a member.
  • Function: To examine reports and accounts of public sector Don’t involve day to day affairs of PSU.
  • The Committee on Petitions
  • Origin: It owes its origin to a resolution moved by a member in the then Council of State on 15 September 1921.
  • Composition: 25 members (15 from Lok Sabha + 10 from Rajya Sabha) Nominated by Speaker/Chairman as the case may be.
  • Function: To estimate every petition referred to it and if the petition complies with the rules to direct that it be circulated

 

Question 7

Which of the following is/are the function/functions of the Cabinet Secretariat?

1. Preparation of agenda for Cabinet Meetings

2. Secretariat assistance to Cabinet Committees

3. Allocation of financial resources to the Ministries

Select the correct answer using the code given below:

(a) 1 only

(b) 2 and 3 only

(c) 1 and 2 only

(d) 1, 2 and 3

Ans: c

Sub-Theme: Executive Functions

  • Statement 1 is correct: A cabinet secretary is appointed for a fixed period of time, usually two years. The All India Services (Death-Cum- Retirement-Benefits) Rules, 1958 state that the government may extend a cabinet secretary’s tenure as long as the overall length of service does not exceed four years. A Cabinet Secretary holds the responsibility of preparation of agenda for Cabinet Meetings.
  • Statement 2 is correct: The prime minister has direct control over the cabinet secretariat. The cabinet secretary, who serves as the ex- officio chairman of the civil services board, is the administrative head of the secretariat. It also gives secretary assistance to the Cabinet Committees.
  • Statement 3 is incorrect: The Secretariat facilitates government decision-making by providing interministerial coordination. The Ministry of Finance, Department of Expenditure is entrusted with the allocation of financial resources to the Ministries.

 

Question 8

The power of the Supreme Court of India to decide disputes between the Centre and the States falls under its:

(a) advisory jurisdiction

(b) appellate jurisdiction

(c) original jurisdiction

(d) writ jurisdiction

Ans: c          

Sub-Theme: Jurisdiction and Powers of Supreme Court

According to Article 131, a dispute between two or more states comes under the original jurisdiction of the Supreme Court. Other disputes which comes under the jurisdiction of the Supreme Court are:

  • State and other states
  • Centre and state on one side and other state on other side
  • Original jurisdiction of the SC extends to any dispute between the GoI and one or more states

 

Question 9

The power to increase the number of judges in the Supreme Court of India is vested in

(a) The President of India

(b) the Parliament

(c) The Chief Justice of India

(d) the Law Commission

Ans: b

Sub-Theme: Constitutional Provisions Pertaining to Supreme Court

  • Article 124(1) mandates that there shall be a Supreme Court of India constituting of a Chief Justice of India and, until Parliament by law prescribes a larger number, of not more than seven other Judges.
  • The power to increase the number of judges in the SC is vested in the Parliament.
  • In the Indian Constitution, Parliament is authorised to regulate judges.
  • In the Supreme Court (Number of Judges) Amendment Bill, 2019, the number of judges in the SC was increased by 3 judges. As a result, there are now 34 judges in total, including the Chief Justice, as opposed to 31

 

Question 10

Which of the following are the discretionary powers given to the Governor of a State?

1. Sending a report to the President of India for imposing the President’s rule

2. Appointing the Ministers

3. Reserving certain bills passed by the State Legislature for consideration of the President of India

4. Making the rules to conduct the business of the State Government

Select the correct answer using the code given below:

(a) 1 and 2 only

(b) 1 and 3 only

(c) 2, 3 and 4 only

(d) 1, 2, 3 and 4

Ans: b

Sub-Theme: Discretionary Power of Governor

Statement 1 is correct: Under Article 356, President’s Rule is imposed if the President, upon receipt of the report from the Governor of the State or otherwise, is satisfied that a situation has arisen in which the government of the State cannot be carried on in accordance with the provisions of the Constitution. This comes under the discretionary power of the Governor.

Statement 2 is incorrect: Article 164 (1) states that the Chief Minister and the other Ministers on the advice of the Chief Minister shall be appointed by the Governor.

Statement 3 is correct: As per Article 200 the Governor of a state can use his legislative power to reserve the bill for the consideration of the President.

Statement 4 is incorrect: Making the rules to conduct the business of the State Government does not come under Governor’s discretionary power. In the Nabam Rebia judgement (2016), the Supreme Court held that the exercise of Governor’s discretion under Article 163 is limited and his choice of action should not be arbitrary or fanciful.

 

Question 11

The sales tax you pay while purchasing a toothpaste is a

(a) Tax imposed by the Central Government

(b) Tax imposed by the central Government but collected by the State Government.

(c) Tax imposed by the State Government but collected by the Central Government.

(d) Tax imposed and collected by the State

Ans: d

Sub-Theme: Financial Relations b/w Centre and State

  • Earlier taxes on toothpaste used to come under CST Act, which is administered by the State Government.
  • The payment of sales Tax was made to the Sales Tax Authority in the state from where the goods are moved.
  • Now the sales taxes on toothpaste comes under GST.

Distribution of Tax Revenues

Article Levy Collection Appropriation Various Taxes
268 Centre State State Stamp duties on shares, cheques, promissory notes, insurance etc.
269 Centre Centre State Taxes on interstate trade and commerce. Revenues do not form part of the consolidated fund of India.
270 Centre Centre Shared between Centre and states All taxes in the union list –income tax (other than agricultural income), corporate tax, etc.
271 Centre Centre Centre Surcharge on taxes under Art 268,269,270.
 

N.A.

 

State

 

State

 

State

Sales tax, excise duty on liquor and Narcotics, octroi, professional tax (max of Rs 2500 – limit kept by constitution)
Article Levy Collection Appropriation Various Taxes

 

Question 12

Which of the following are associated with ‘Planning’ in India?

1. The Finance Commission

2. The National Development Council

3. The Union Ministry of Rural Development

4. The Union Ministry of Urban Development

5. The Parliament

Select the correct answer using the code given below:

(a) 1, 2 and 5 only

(b) 1, 3 and 4 only

(c) 2 and 5 only

(d) 1, 2, 3, 4 and 5

Ans: c         

Sub-Theme: NDC / Planning Commission / NITI Ayog

  • Statement 1 is incorrect: It is constituted by the President under Article 280 to give its recommendations on the distribution of tax revenues between the Union and the States. It defines the financial relations between the Centre and the States.
  • Statement 2 is correct: Earlier, NDC served as “the highest decision-making authority” in the country on development matters. NDC used to advise the Planning Commission on planning. Statement 3 is incorrect: The Union Ministry of Rural Development is entrusted with the task of accelerating the socio-economic development of rural India.
  • Statement 4 is incorrect: The Union Ministry of Urban Development is entrusted with the formulation and administration of the rules and regulations and laws relating to the housing and urban development in India.
  • Statement 5 is correct: Parliament is the supreme legislative institution of the Republic of India. It is directly responsible to make laws for the welfare of the society.

 

2013

 

Question 1

In the context of India, which of the following principles is/are, implied institutionally in the parliamentary government?

1. Members of the Cabinet are Members of the Parliament.

2. Ministers hold the office till they enjoy confidence in the Parliament.

3. The Cabinet is headed by the Head of the State.

Select the correct answer using the code given below:

(a) 1 and 2 only

(b) 3 only

(c) 2 and 3 only

(d) 1, 2 and 3

Ans: a

Sub-Theme: Forms of Government

Parliamentary System entails the Executive responsible to the legislature for its policies and legislations. The Constitution of India provides for a parliamentary form of government, both at the Centre and in the States. The Parliamentary Government is also known as Cabinet Government/Responsible Government/Westminster Model. Some of its features are as follows:

  • In the Parliamentary System of Government members of the Cabinet are members of the Parliament. (Statement 1 is correct).
  • The ministers in the Parliament hold the office till they enjoy the confidence of the Parliament. (Statement 2 is correct)
  • The Head of the state is President, and he/she does not head the cabinet, he is considered to be the nominal executive. (Statement 3 is incorrect)

 

Question 2

Economic Justice’ as one of the objectives of the Indian Constitutional has been provided in:

(a) the Preamble and the Fundamental Rights

(b) the Preamble and the Directive Principles of State Policy

(c) the Fundamental Rights and the Directive Rights and the Directive Principles of State Policy

(d) None of the above

Ans: b

Sub-Theme: Key Words in the Preamble and Socialistic principles of DPSP

“Economic Justice” as one of the objectives of the Indian Constitution has been provided both in the Preamble of the Constitution and the Directive Principles of State Policy (DPSP). This is evident as follows:

Preamble:

  • The ideal of ‘justice’ mentioned in the preamble embraces three distinct forms- social, economical and political, where economic justice denotes the non- discrimination between people on the basis of economic factors.

Directive Principles of State Policy (DPSP):

  • The Socialistic principles of DPSP reflect the ideology of socialism. It aims to provide social and economic justice and set the path towards a welfare state.
  • The 44th Amendment Act of 1978 added the Article 38 in DPSP, which requires the State to minimise inequalities in income, status, facilities and opportunities.

 

Question 3

According to the Constitution of India, which of the following are fundamental for the governance of the country? 

(a) Fundamental Rights

(b) Fundamental Duties

(c) Directive Principles of State Policy

(d) Fundamental Rights and Fundamental Duties

Ans: c

Sub-Theme: Features of the Directive Principles

  • According to Article 37 of the Indian Constitution, though Directive Principles of State Policy (DPSP) are not enforceable by any court, but the principles therein laid down are nevertheless fundamental in the governance of the country and it shall be the duty of the State to apply these principles in making laws.
  • DPSP constitutes a comprehensive socio economic and political programme for a modern democratic State. DPSP are ‘positive in nature’, which embodies the concept of a ‘welfare state’ and not that of a ‘police state’. DPSP are the constitutional instructions or recommendations to the State in legislative, executive and administrative matters.

Additional Information:

Fundamental Rights and Duties are related to the role, responsibility and rights of the people (citizens/aliens).

 

Question 4

Consider the following statements:

1. An amendment to the Constitution of India can be initiated by an introduction of a bill in the Lok Sabha only.

2. If such an amendment seeks to make changes in the federal character of the Constitution, the amendment also requires to be ratified by the legislature of all the States of India.

Which of the statements given above is/are correct?

(a) 1 only

(b) 2 only

(c) Both 1 and 2

(d) Neither 1 nor 2

Ans: d

Sub-Theme: Procedure for the Amendment

Statement 1 is incorrect: An amendment of the Constitution can be initiated by the introduction of a bill in either house of the Parliament.

Statement 2 is incorrect: If the bill seeks to amend the Federal provisions of the Constitution, then it must also be ratified by the legislatures of half of the states by a simple majority

NOTE: In this question focus on extreme/ absolute words such as “Only” in sentence 1 and “All” in sentence 2 to pinpoint your thinking on given statements. Not all the time, but most of the time such reasoning will help you out.

 

Question 5

Consider the following statements:

1. The Chairman and the Deputy Chairman of the Rajya Sabha are not the members of that house.

2. While the nominated members of the two Houses of the Parliament have no voting right in the presidential election, they have the right to vote in the election of the Vice President.

Which of the statements given above is/are correct?

(a) 1 only

(b) 2 only

(c) Both 1 and 2

(d) Neither 1 nor 2

Ans: b

Sub-Theme: Election of the Vice President

  • Statement 1 is incorrect: In accordance with Article 64 of the Constitution of India, the Vice President of India is the ex-officio Chairman of Rajya Sabha. The House also elects a Deputy Chairman from among its members. Hence, the Deputy Chairman of the Rajya Sabha is the member of that house.
  • Statement 2 is correct: Vice President elected indirectly by an electoral college consisting of all members of parliament (Both elected & nominated). The election is conducted in accordance with the system of proportional representation by means of single transferable vote. State government does not participate in the Vice President’s election.

 

Question 6

Consider the following statements:

The Parliamentary Committee on Public Accounts:

1. Consists of not more than 25 Members of the Lok Sabha.

2. Scrutinises appropriation and finance accounts of the Government.

3. Examines the report of the Comptroller and Auditor General of India.

Which of the statements given above is/are correct?

(a) 1 only

(b) 2 and 3 only

(c) 3 only

(d) 1, 2 and 3

Ans: b         

Sub-Theme: Parliamentary Committee

Statement 1 is incorrect: Parliamentary Committee on Public Accounts consists of 22 members of which 15 are from Lok Sabha and 7 from Rajya Sabha. It is formed every year with its members serving one-year terms in office.

Statement 2 is correct: It scrutinises appropriation and finance accounts of the Government.

Statement 3 is correct: It’s chief function is to examine the report of the Comptroller and Auditor General of India.

 

Question 7

Consider the following statements:

1. The Council of Ministers in the Centre shall be collectively responsible to the Parliament.

2. The Union Ministers shall hold the office during the pleasure of the President of India.

3. The Prime Minister shall communicate to the President about the proposals for legislation.

Which of the statements given above is/are correct?

(a) 1 only

(b) 2 and 3 only

(c) 1 and 3 only

(d) 1, 2 and 3

Ans: b     

Sub-Theme: Functions of Parliament

Statement 1 is incorrect: As per Article 75(3) of the constitution “the Council of Ministers shall be collectively responsible to the House of the People” not to the whole parliament as it includes Rajya Sabha and Lok Sabha both.

Statement 2 is correct: Article 75(2) of the Constitution of India mandates that the Ministers shall hold office during the pleasure of the President.

Statement 3 is correct: According to Article 78, it is the duty of the Prime Minister to communicate to the President all decisions of the Council of Ministers relating to the administration of the affairs of the union and proposals for legislation.

 

Question 8

What will follow if a Money Bill is substantially amended by the Rajya Sabha?

(a) The Lok Sabha may still proceed with the Bill, accepting or not accepting the recommendations of the Rajya Sabha

(b) The Lok Sabha cannot consider the Bill further

(c) The Lok Sabha may send the Bill to the Rajya Sabha for reconsideration

(d) The President may call a joint sitting for passing the Bill

Ans: a

Sub-Theme: Money Bill

Money Bill (Article 110):

  • Matter concerned with financial matters like the imposition, abolition, remission, alteration or regulation of any tax.
  • Can be introduced Only in Lok
  • Introduced only on the recommendation of the President.
  • Cannot be amended or rejected by the Rajya Sabha should return the bill with or without recommendation Lok Sabha may accept or reject recommendation.
  • Rajya Sabha can keep it for a maximum period of 14 days.
  • Requires the certification of the
  • Rajya Sabha has limited power with respect to money bills so no chance of any
  • The President can reject or approve but cannot return it to parliament for

 

Question 9

The Parliament can make any law for whole or any part of India for implementing international treaties:

(a) with the consent of all the States

(b) with the consent of the majority of States

(c) with the consent of the States concerned

(d) without the consent of any State

Ans: (d)

Sub-Theme: Functions of Parliament

Legislative Power

  • The Parliament is empowered to make laws on all matters listed in the Union List and the Concurrent List of the Constitution.
  • As per Article 250, it can also formulate laws over the State List when an Emergency is declared in the country, or President’s Rule is declared in any State.
  • Article 253 says that it can also formulate laws on any matter pertaining to the State List if it is deemed necessary for the implementation of international treaties or agreements concluded with foreign nations.

 

Question 10

With reference to National Legal Services Authority consider the following statements:

1. Its objective is to provide free and competent legal services to the weaker section of the society on the basis of equal opportunity.

2. It issues guidelines for the State Legal Services Authorities to implement the legal programmes and schemes throughout the country.

Which of the statements given above is/are correct?

(a) 1 only

(b) 2 only

(c) Both 1 and 2

(d) Neither 1 nor 2

Ans: c

Sub-Theme: Legal Services Authority/Legal Aid

Statement 1 is correct: The objective of National Legal Services Authority (NALSA) is to provide free legal services to weaker sections of society for securing justice and are not denied to any citizen by reasons of economic or other disabilities.

Statement 2 is correct: In every State, there is a State Legal Service Authority to give effect to the policies and directions of the NALSA and to give free legal services to the people and conduct Lok Adalats in the State.

 

Question 11

Which one of the following statements is correct?

(a) In India, the same person cannot be appointed as Governor for two or more States at the same time.

(b) The Judges of the High Court of the States in India are appointed by the Governor of the State just as the Judges of the Supreme Court are appointed by the President.

(c) No procedure has been laid down in the Constitution of India for the removal of a Governor from his/her post.

(d) In the case of a Union Territory having a legislative setup, the Chief Minister is appointed by the Lt. Governor on the basis of majority support.

Ans: c

Sub-Theme: Appointment of Governor/ Administration of Union Territories

Articles 153 to 167 of Part VI of the Indian Constitution deals with the provisions relating to the Governor of the States. The Governor of the state is the Chief executive head of the state. He is also the constitutional (nominal) head. He is the agent of the Central Government. The Indian Constitution does not lay down any grounds upon which a governor may be removed by the President.

Additional Information:

  • Usually, there is a governor for each state, but the 7th Constitutional Amendment Act of 1956 facilitated the appointment of the same person as a governor for two or more states. The 7th Constitutional Amendment also provided for the:
    • Appointment of additional and acting judges of the high court.
    • Establishment of a common high court for two or more states;
    • Extension the jurisdiction of high courts to union territories.
  • As per Article 217(1) the Chief Justice and Judges of the High Courts are to be appointed by the President.
  • Section 45 in The Government of Union Territories Act, 1963 states that the CM of the UT(Delhi, Puducherry and Jammu and Kashmir) is appointed by the President of India, while the oath is administered by the Lt. Governor of the respective UT.

 

Question 12

The Government enacted the Panchayat Extension to Scheduled Areas (PESA) Act in 1996. Which one of the following is not identified as its objective?

(a) To provided self-governance

(b) To recognize traditional rights

(c) To create autonomous regions in tribal areas

(d) To free tribal people from exploitation

Ans: c          

Sub-Theme: PESA Act

Option (c) is correct: The PESA act is for 5th schedule areas whereas creation of autonomous regions in tribal areas comes under ambit of 6th schedule.

 

Question 13

Under the Scheduled Tribes and Other Traditional Forest Dwellers (Recognition of Forest Rights) Act, 2006, who shall be the authority to initiate the process for determining the nature and extent of individual or community forest rights or both?

(a) State Forest Department

(b) District Collector/Deputy Commissioner

(c) Tahsildar/Block Development Officer/Mandal Revenue Officer

(d) Gram Sabha

Ans: d      

Sub-Theme: Power of Gram Sabha

Forest Rights Act, 2006:

  • The act recognizes and vests the forest rights and occupation in Forest land to forest Dwelling Scheduled Tribes (FDST) and Other Traditional Forest Dwellers (OTFD) who have been residing in such forests for generations.
  • The act further enjoins upon the Gram Sabha and rights holders the responsibilities and authority for sustainable use, conservation of biodiversity and maintenance of ecological balance of FDST and OTFD.
  • While protecting the FDST and OTFD’s way of life and food security, it strengthens the conservation regime for the forests.
  • It aims to rectify the historical wrongs committed against the OTFD and FDST, who are crucial to the sustainability and survival of the forest ecosystem.

 

Question 14

Consider the following statements:

Attorney General of India can-

1. take part in the proceedings of the Lok Sabha

2. be a member of a committee of the Lok Sabha

3. speak in the Lok Sabha

4. vote in the Lok Sabha

Which of the statements given above is/are correct?

(a) 1 only

(b) 2 and 4

(c) 1, 2 and 3

(d) 1 and 3 only

Ans: (c)          

Sub-Theme: Attorney General of India

Article 76 of the Constitution of India deals with the provisions of the Attorney General of India. The Attorney General of India is appointed by the President of India. He is a Qualified to be Judge of Supreme Court (i.e. Citizen of India and must have been a judge of some high court for 5 years or an advocate of some high court for 10 years) or an eminent jurist, in the opinion of the president.

He has the Right to speak and to take part in the proceedings of Parliament or their joint sitting and any committee of the Parliament of which he is a member, but without a right to vote.

 

Question 15

Consider the following statements:

1. The National Development Council is an organ of the Planning Commission.

2. The Economic and Social Planning is kept in the Concurrent List in the Constitution of India.

3. The Constitution of India prescribes that Panchayats should be assigned the task of preparation of plans for economic development and social justice.

Which of the statements given above is/are correct?

(a) 1 only

(b) 2 and 3 only

(c) 1 and 3 only

(d) 1, 2 and 3

Ans: b        

Sub-Theme: NDC / Planning Commission / NITI AAYOG

Statement 1 is correct: National Development Council (NDC) and Planning Commission (PC) have been abolished now. NDC was an advisory body and used to advise the Planning Commission on planning and included all CMs and union ministers etc.

Statement 2 is correct: The Economic and Social Planning is kept in the Concurrent List in the Constitution of India.

Statement 3 is correct: Preparation of plans and implementation of plans for economic development and social justice are one of the major powers and functions that comes under Panchayat Raj Institution.

 

Question 16

Who among the following constitute the National Development Council?

1. The Prime Minister

2. The Chairman, Finance Commission

3. Ministers of the Union Cabinet

4. Chief Ministers of the States

Select the correct answer using the codes given below.

(a) 1, 2 and 3 only

(b) 1, 3 and 4 only

(c) 2 and 4 only

(d) 1, 2, 3 and 4

Ans: b        

Sub-Theme: NDC/Planning Commission/NITI AAYOG

Option (b) is correct: NDC comprises the Prime Minister, all the Union Cabinet Ministers, the Chief Ministers of all States and Union Territories and the Members of the Planning Commission. It does not include the chairman of the Finance Commission.

 

Question 17

Which of the following bodies does not/do not find mention in the Constitution?

1. National Development Council

2. Planning Commission

3. Zonal Councils

Select the correct answer using the codes given below:

(a) 1 and 2 only

(b) 2 only

(c) 1 and 3 only

(d) 1, 2 and 3

Ans: d         

Sub-Theme: NDC / Planning Commission / NITI Ayog

The National Development Council, the Planning Commission and the Zonal Councils do not find its mention in the Constitution. They are the Non-Constitutional Bodies. Few details about the same are as follows:

  • The National Development Council (NDC) was set up on 6 August 1952 by an executive order of the government. It served as “the highest decision-making authority” in the country on development matters. The National Development Council used to advise the Planning Commission on planning. Currently, there is no existence of NDC and Planning Commission, it has been replaced by NITI AAYOG.
  • The Planning Commission of India was set up by a Resolution of the Government of India in March 1950.
  • The Zonal Councils are the statutory (and not the constitutional) bodies. They are established by an Act of the Parliament, that is, the States Reorganisation Act of 1956. The act divided the country into five zones- Northern, Central, Eastern, Western and Southern and provided a zonal council for each zone.

 

2012

 

Question 1

The distribution of powers between the Centre and the States in the Indian Constitution is based on the scheme provided in which of the following:

(a) Morley-Minto Reforms, 1909

(b) Montagu-Chelmsford Act, 1919

(c) Government of India Act, 1935

(d) Indian Independence Act, 1947

Ans: c

Sub-Theme: Acts and Regulations under The Crown Rule (1858–1947)

The distribution of powers between the Centre and the States in the Indian Constitution is given under 7th Schedule of the Indian Constitution. The 7th Schedule divides the powers into 3 lists: Union List, State List and Concurrent List. This division of powers is based on the Government of India Act, 1935. It divided the powers between the Centre and provinces in terms of three lists – Federal List (for Centre, with 59 items), Provincial List (for provinces, with 54 items) and the Concurrent List (for both, with 36 items).

Other Important Features of Government of India Act, 1935:

  • It abolished dyarchy in the provinces and introduced ‘provincial autonomy’ in its place, while adopting the dyarchy at the
  • It introduced Bicameralism in six out of eleven provinces.
  • It further extended the principle of communal representation to the depressed classes (Scheduled Castes), women and labour (workers).

It also abolished the Council of India, established by the Government of India Act of 1858.

 

 

Question 2

Consider the following provisions under the Directive Principles of State Policy as enshrined in the Constitution of India:

1. Securing for citizens of India a uniform civil code

2. Organizing village panchayat

3. Promoting cottage industries in rural areas

4. Securing for all the workers reasonable leisure and cultural opportunities.

Which of the above are the Gandhian Principles that are reflected in the Directive Principles of State Policy?

(a) 1, 2 and 4 only

(b) 2 and 3 only

(c) 1, 3 and 4 only

(d) 1, 2, 3 and 4

Ans: b

Sub-Theme: Gandhian Principles and Liberal- Intellectual Principles

  • The Directive Principles of State Policy (DPSP) are mentioned under Articles 36 to 51 of the Indian Constitution. The DPSPs have been borrowed from the Constitution of Ireland.
  • Though the Indian Constitution does not specify classification of Directive Principles of State Policy, but on the basis of content, these are classified into Socialist, Gandhian and Liberal-Intellectual.
  • Article 40 of the Constitution deals with Organising Village Panchayats and endows them with necessary powers and authority to enable them to function as units of self-government. This is one of the Gandhian Principles in DPSP.
  • The idea of Promoting Cottage Industries on an individual or co-operation basis in rural areas is based on Gandhian Principles and the same is mentioned under Article 43.

Additional Information:

  • Article 44 one of the directive principles of the Constitution deals with securing for citizens of India a Uniform Civil Code (UCC) for the citizens throughout the territory of India. This article comes under the Liberal- Intellectual Principles, which represent the ideology of A UCC is one that would provide for one law for the entire country, applicable to all religious communities in their personal matters such as marriage, divorce, inheritance, adoption etc.
  • This section of Article 43 comes under the Socialistic Principles of DPSP which says, to secure a living wage, a decent standard of life and social and cultural opportunities for all workers.

 

Question 3

Which of the following provisions of the Constitution of India have a bearing on Education? 

1. Directive Principles of State Policy

2. Rural and Urban Local Bodies

3. Fifth Schedule

4. Sixth Schedule

5. Seventh Schedule

Select the correct answer using the codes given below:

(a) 1 and 2 only

(b) 3, 4 and 5 only

(c) 1, 2 and 5 only

(d) 1, 2, 3, 4 and 5

Ans: d

Sub-Theme: Combination of DPSP and schedules Various provisions in the Constitution of India have a bearing on These include Directive Principles of State Policy, Rural and Urban Local Bodies and Seventh Schedule of the Constitution:

Directive Principle of State Policy (DPSP):

  • Under Article 45 of Liberal-Intellectual Principles lays down the provision of early childhood care and education for all children until they complete the age of six
  • With a view of making the right to free and compulsory education a fundamental right, the 86th Amendment Act of 2002 changed the subject-matter of Article 45 and made elementary education a fundamental right under Article 21 A, conferring on all children in the age group of 6 to 14
  • The Act amends in Part-III, Part–IV and Part-IV(a) of the Constitution.

Rural and Urban Local Bodies:

  • The Schedule 11 added by the 73rd Constitutional Amendment Act of 1992, specifies the powers, responsibilities and authority of Panchayat and under Article 243-G the Panchayati Raj Institutions have allowed 29 subjects to work in their local areas and Education was one of them.
  • The Schedule 12 added by the 74th Constitutional Amendment Act of 1992, specifies the powers, responsibilities and authority of Municipalities and under the Article 243-W the Constitution provides for 18 subjects to the Municipalities and education is included here too.
  • Fifth and Sixth Schedules are related to Scheduled and Tribal regions, which also deal with education.
  • Seventh Schedule: The Seventh Schedule of the Constitution contains three lists- Union List, State List and Concurrent List.
  • The 42nd Amendment Act, 1976 shifted the following subjects from State list to Concurrent list –
    • Education
    • Forests
    • Weights & Measures
    • Protection of Wild Animals and Birds
    • Administration of Justice

 

Question 4

Which of the following is/are among the Fundamental Duties of citizens laid down in the Indian Constitution?

1. To preserve the rich heritage of our composite culture.

2. To protect the weaker sections from social injustice.

3. To develop the scientific temper and spirit of inquiry

4. To strive towards excellence in all spheres of individual and collective activity.

Select the correct answer using the codes given below:

(a) 1 and 2 only

(b) 2 only

(c) 1, 3 and 4 only

(d) 1, 2, 3 and 4

Ans: c

Sub-Theme: List of Fundamental Duties

  • Article 51A of part IV-A of the Indian Constitution deals with the Fundamental Duties. These duties are the moral and civic obligations that all citizens of a nation
  • In 1976, the Fundamental Duties were first recommended by the Swaran Singh Committee, its need was felt during the internal emergency (1975-77) and were added to the Constitution by 42nd Constitutional Amendment Act, 1976.
  • Later, one more Fundamental Duty was added by 86th Constitutional Amendment Act, 2002.

List of Fundamental Duties are –

  • To abide by the Constitution and respect National Flag and National Anthem (Not national song)
  • To follow the noble ideals that inspired the struggle for freedom.
  • To protect the sovereignty, unity and integrity of India.
  • To render national service when called upon to do so.
  • To promote the common spirit of brotherhood amongst all Indians and to renounce practices derogatory to the dignity of women.
  • To preserve the rich heritage of the country’s culture.
  • To protect the national environment including forests, lakes, wildlife, etc.
  • To develop scientific temper, humanism and spirit of reform and inquiry.
  • To safeguard public property and to abjure
  • To strive towards excellence in all spheres of individuals.
  • To provide opportunity for education to his child between the age 6 and 14 years

Thus, Protecting the weaker sections from social injustice is more relevant to Directive Principle of State Policy (DPSP), not Fundamental Duties.

 

Question 5

According to the Constitution of India, it is the duty of the President of India to cause to be laid before the Parliament which of the following?

1. The Recommendation of the Union Finance Commission

2. The Report of the Public Accounts Committee.

3. The Report of the Comptroller and Auditor General

4. The Report of the National Commission for Scheduled Castes.

Select the correct answer using the codes given below:

(a) 1 only

(b) 2 and 4 only

(c) 1, 3 and 4 only

(d) 1, 2, 3 and 4

Ans: c

Sub-Theme: Legislative Powers of President

India follows a Parliamentary  type of Government in which the President is the constitutional head of the state. He is the Executive head of India, and all the executive powers of the Union shall be vested in him. The President lays Reports and Statements before the parliament of the following bodies:

  • Reports of the Comptroller and Auditor General
  • Annual report of UPSC
  • Reports of Finance
  • Reports of Special officers of SC & ST
  • Report of the National Commission for Scheduled Castes and Scheduled Tribes
  • Report of the National Commission of Backward Classes.

 

Question 6

The Prime Minister of India, at the time of his/her appointment:

(a) Need not necessarily be a member of one of the Houses of the Parliament but must become a member of one of the Houses within six months

(b) Need not necessarily be a member of one of the Houses of the Parliament but must become a member of the Lok Sabha within six months

(c) Must be a member of one of the Houses of the Parliament

(d) Must be a member of the Lok Sabha

Ans: a

Sub-Theme: Appointment of PM

    • As per the article 75 (1) of the Indian Constitution, the Prime Minister is appointed by the President, who also appoints other ministers on the advice of the Prime
    • A person who is not a member of the Parliament/State legislature can be appointed as PM/CM for six months, within which time, he should be elected to the Parliament/State legislature. This is provided by a Supreme Court in one of the

 

Question 7

In the Parliament of India, the purpose of an adjournment motion is

(a) to allow a discussion on a definite matter of urgent public importance.

(b) to let opposition members, collect information from the ministers.

(c) to allow a reduction of specific amounts in demand for grants.

(d) to postpone the proceedings to check the inappropriate or violent behaviour on the part of some members.

Ans: a

Sub-Theme: Devices of Parliamentary Proceedings

  • Adjournment Motion: Adjournment Motion is one of the devices of Parliamentary It is introduced to draw attention of the House to a definite matter of urgent public importance which is a definite, factual, urgent matter of recent occurrence.

Additional Information:

  • Question Hour: In the First Hour of parliamentary sitting usually members ask questions and ministers give answers but sometimes questions can be asked to private members too.
  • Three types of Question are asked namely:
    1. Starred Question: Require oral answer and Supplementary question can be
    2. Unstarred Question: Require written answer and Supplementary question cannot be asked.
    3. Short Notice Question: Answered orally and asked on short notice of less than 10 days.

 

Question 8

Consider the following statements:

1. Union Territories are not represented in the Rajya sabha.

2. It is within the purview of the Chief Election Commissioner to adjudicate the election disputes.

3. According to the Constitution of India, the Parliament consists of the Lok Sabha and the Rajya Sabha only.

Which of the statements given above is/are correct?

(a) 1 only

(b) 2 and 3

(c) 1 and 3

(d) None of these

Ans: d

Sub-Theme: Basic Provisions of Parliament

  • Statement 1 is incorrect: Union Territories like Delhi and Pondicherry are represented in the Rajya Sabha. Also, The union territory will get its representation as soon as the Union Territory of Jammu and Kashmir Legislative Assembly is elected.
  • Statement 2 is incorrect: The Supreme Court and High Court adjudicate on the election disputes. It is not within the purview of the Chief Election Commissioner to adjudicate election disputes. As per article 323 (b) of the Constitution of India – to resolve all matters related to election disputes of parliament or state legislature, union parliament can constitute a tribunal. It also excludes judiciary jurisdiction to look into these matters.
  • Statement 3 is incorrect: According to the Constitution of India, the Parliament consists of the Lok Sabha, the Rajya Sabha and the President of India.

 

Question 9

Regarding the office of the Lok Sabha Speaker, consider the following statements:

1. He/she holds the office during the pleasure of the president.

2. He/she need not be a member of the House at the time of his/her election but has to become a member of the House within six months from the date of his/her election.

3. If he/she intends to resign, the letter of his/her resignation has to be addressed to the Deputy speaker.

Which of the statements given above is/are correct?

(a) 1 and 2 only

(b) 3 only

(c) 1, 2 and 3

(d) None of these

Ans: b

Sub-Theme: Speaker of Lok Sabha

  • Statement 1 is incorrect: The Speaker is the presiding officer or the Chairman of the Lok Sabha. Article 93 to 97 of the Constitution of India deals with the provisions of Presiding officer of the Lok Sabha. Usually, the Speaker remains in office during the life of the Lok Sabha.
  • Statement 2 is incorrect: Elected by the Lok Sabha from amongst its members and the date of election of the Speaker of the Lok Sabha is fixed by the President.
  • Statement 3 is correct: The Speaker of the Lok Sabha has to vacate his office earlier in any of the following three cases:
    • If he ceases to be a member of the Lok Sabha;
    • If he resigns by writing to the Deputy Speaker; and
    • If he is removed by a resolution passed by a majority of all, then members of the Lok Sabha. Such a resolution can be moved only after giving 14 days’ advance notice.

 

Question 10

Which of the following special powers have been conferred on the Rajya Sabha by the Constitution of India?

(a) To change the existing territory of a State and to change the name of a State.

(b) To pass a resolution empowering the Parliament to make laws in the State List and to create one or more All India Services.

(c) To amend the election procedure of the President and to determine the pension of the President after his/her retirement.

(d) To determine the functions of the Election Commission and determine the number of Election Commissioners.

Ans: b

Sub-Theme: Special Power of Rajya Sabha

The Rajya Sabha has been given four exclusive or special powers that are not enjoyed by the Lok Sabha:

  • It can authorise the Parliament to make a law on a subject enumerated in the State List (Article 249).
  • It can authorise the Parliament to create new All-India Services common to both the Centre and states (Article 312).
  • Resolution for the removal of the vice- president can be introduced only in the Rajya Sabha not in the Lok Sabha (Article 67).
  • A proclamation for imposing national emergency or president’s rule or financial emergency (Articles 352, 356 and 360) can remain effective even if it is approved by the Rajya Sabha alone if Lok Sabha is dissolved before or during the emergency period.

 

Question 11

A deadlock between the Lok Sabha and the Rajya Sabha calls for a joint sitting of the Parliament during the passage of

1. Ordinary Legislation

2. Money Bill

3. Constitution Amendment Bill

Select the correct answer using the codes given below:

(a) 1 only

(b) 2 and 3 only

(c) 1 and 3 only

(d) 1, 2 and 3

Ans: a

Sub-Theme: Joint Session of Parliament

Joint sitting (Article 108) of two house:

  • The Constitution offers extraordinary mechanisms to break a deadlock between the two Houses over the passage of a bill.
  • Summon: The president can summon joint sitting for the purpose of deliberating and voting on the bill.
  • Bills: Applicable to ordinary bills or financial bills only and not to money bills or Constitutional amendment bills.
  • Quorum: The quorum to constitute a joint sitting shall be one-tenth of the total number of members of the Houses.
  • Presides: The Speaker of Lok Sabha presides over a joint sitting and the Deputy Speaker, in his absence. Chairman of Rajya Sabha does not preside over a joint sitting as he is not a member of either House of Parliament but deputy chairman of Rajya Sabha can.

 

Question 12

Which reference to the Delimitation Commission, consider the following statements:

1. The orders of the Delimitation Commission cannot be challenged in a Court of Law.

2. When the orders of the Delimitation Commission are laid before the Lok Sabha or State Legislative Assembly, they cannot effect any modifications in the orders.

Which of the statements given above is/are correct?

(a) 1 only

(b) 2 only

(c) Both 1 and 2

(d) Neither 1 nor 2

Ans: c

Sub-Theme: Delimitation

Statement 1 is correct: The act of Delimitation is redrawing boundaries of Lok Sabha and Assembly constituencies to represent changes in population and done on the basis of the preceding Census. Its objective is to ensure that all population groups are fairly represented, with equitable geographic distribution so that no political party benefits. A Delimitation Commission is in charge of carrying out this exercise, and its decisions are binding and cannot be challenged in court.

Statement 2 is correct: When the orders of the Delimitation Commission are laid before the Lok Sabha or State Legislative Assembly, they cannot effect any modifications in the orders. Additional Information:

  • The Delimitation Commission is appointed by the President of India and works in partnership with the Election Commission of India.
  • Members: A serving or retired Supreme Court judge, Chief Election Commissioner or an Election Commissioner nominated by CEC and Election Commissioners of the respective state.
  • Function: To determine the number and boundaries of constituencies, to identify seats reserved for SC/ST.
  • Power: It is a high power body whose orders have the force of law and cannot be called in question before any court.
  • Delimitation Commissions have been set up four times: 1952, 1963, 1973 and 2002 under the Acts of 1952, 1962, 1972 and 2002.

 

Question 13

Which of the following are the methods of Parliamentary control over public finance in India?

1. Placing Annual Financial Statement before the Parliament.

2. Withdrawal of money from Consolidated Fund of India only after passing the Appropriation Bill.

3. Provisions of supplementary grants and vote-on accounts.

4. A periodic or at least a mid-year review of programme macroeconomic forecasts and expenditure by a Parliamentary Budget Office.

5. Introducing the Finance Bill in the Parliament.

Select the correct answer using the codes given below:

(a) 1, 2, 3 and 5 only

(b) 1, 2 and 4 only

(c) 3, 4 and 5 only

(d) 1, 2, 3, 4 and 5

Ans: a        

Sub-Theme: Union Budget/Annual Financial Statement

  • Statement 1 is correct: Article 112 of the Indian Constitution deals with placing annual financial statements in Parliament.
  • Statement 2 is correct: Article 114 mandates that the government can withdraw money from it only after receiving approval from Parliament.
  • Statement 3 is correct: Votes on account, votes on credit and exceptional grants are specified in Article 116 of the constitution.
  • Statement 4 is incorrect: There is no such Parliamentary budget office to review programs of Government. Though there is an Estimate Committee of parliament to review policies and continuous examination of estimates from time to time throughout the year.
  • Statement 5 is correct: The Finance Bill is a part of the Union Budget, stipulating all the legal amendments required for the changes in taxation proposed by the Finance Minister.

 

Question 14

Which of the following are included in the original jurisdiction of the Supreme Court?

1. A dispute between the Government of India and one or more States.

2. A dispute regarding elections to either House of the Parliament or that of Legislature of a State.

3. A dispute between the Government of India and a Union Territory.

4. A dispute between two or more

Select the correct answer using the codes given below:

(a) 1 and 2

(b) 2 and 3

(c) 1 and 4

(d) 3 and 4

Ans: c        

Sub-Theme: Jurisdiction and Powers of Supreme Court

  • Statement 1 is correct: According to Article 131, a dispute between the Government of India and one or more States.
  • Statement 2 is incorrect: As per Article 329 (b), no election to either House of Parliament or to the House or either House of the Legislature of a State shall be called in question except by an election petition presented to such authority and in such manner as may be provided for by or under any law made by the appropriate Legislature.
  • Statement 3 is incorrect: Article 131 doesn’t mention UT anywhere. Further, the original jurisdiction mentioned in 131 is regarding issues between the federal units; UTs being part of the Central Government in one way or the other.
  • Statement 4 is correct: According to Article 131, a dispute between two or more states comes under the original jurisdiction of the Supreme Court.

 

Question 15

What is the provision to safeguard the autonomy of the Supreme Court of India?

1. While appointing the Supreme Court Judges, the President of India has to consult the Chief Justice of India.

2. The Supreme Court Judges can be removed by the Chief Justice of India only.

3. The salaries of the Judges are charged on the Consolidated Fund of India to which the legislature does not have to vote.

4. All appointments of officers and staff of the Supreme Court of India are made by the Government only after consulting the Chief Justice of India.

Which of the statements given above is/are correct?

(a) 1 and 3 only

(b) 3 and 4 only

(c) 4 only

(d) 1, 2, 3 and 4

Ans: a

Sub-Theme: Independence of Supreme Court

  • Statement 1 is correct: The judges of the SC are appointed by the President(which means the cabinet) in consultation with the members of the judiciary itself (i.e. judges of the SC and the HC).
  • Statement 2 is incorrect: SC judge can be removed from his office by an order of President. The President can order for removal of SC Judge only after an address by Parliament has been presented to him in the same session for such removal.
  • Statement 3 is correct: The salaries, allowances and pensions of the judges and the staff as well as all the administrative expenses of the SC are charges on the Consolidated Fund of India. Thus, they are non-votable by the Parliament.
  • Statement 4 is incorrect: The Chief Justice of India can appoint officers and servants of the SC without any interference from the executive. He can also prescribe their conditions of service.

 

Question 16

In the areas covered under the Panchayat (Extension to the Scheduled Areas) Act, 1996, what is the role/ power of Gram Sabha?

1. Gram Sabha has the power to prevent alienation of land in the Scheduled Areas.

2. Gram Sabha has the ownership of minor forest produce.

3. Recommendation of Gram Sabha is required for granting prospecting licence or mining lease for any mineral in the Scheduled Areas.

Which of the statements given above is/are correct?

(a) 1 only

(b) 1 and 2 only

(c) 2 and 3 only

(d) 1, 2 and 3

Ans: b

Sub-Theme: PESA Act

  • Statement 1 is correct: Gram Sabha has the power to prevent alienation of land and to restore any unlawfully alienated land of a scheduled tribe.
  • Statement 2 is correct: The Gram Sabha, in the PESA Act, was granted wide ranging powers starting from consultation on land acquisition to that of ownership over minor forest products and leasing of minor minerals.
  • Statement 3 is incorrect: Gram Sabha can grant prospecting license for mining lease for minor minerals and concessions for such activities and not all/any minerals.
NOTE: UPSC has replaced words “minor mineral” by “any mineral” to create confusion.

 

Question 17

In India, other than ensuring that public funds are used efficiently and for the intended purposes, what is the importance of the office of the Comptroller and Auditor General (CAG)?

1. CAG exercises exchequer control on behalf of the Parliament when the President of India declares a national emergency/financial emergency.

2. CAG reports on the execution of projects or programmes by the ministries are discussed by the Public Accounts Committee.

3. Information from CAG reports can be used by investigating agencies to press charges against those who have violated the law while managing public finances.

4. While dealing with the audit and accounting of government companies, CAG has certain judicial powers for prosecuting those who violate the law.

Which of the statements given above is/are correct?

(a) 1, 2 and 4 only

(b) 2 only

(c) 2 and 3 only

(d) 1, 2, 3 and 4

Ans: c          

Sub-Theme: CAG

  • Statement 1 is incorrect: During the Financial Emergency, Executive authority of Centre extends to directing any state to observe such canons of financial propriety as are specified by it. Similarly, during national emergency the Parliament may extend the normal tenure of a state Legislative Assembly by one year each time, subject to a maximum period of six months after the emergency has ceased to operate.
  • Statement 2 is correct: PAC’s chief function is to examine the audit report of Comptroller and Auditor General (CAG) after it is laid in the Parliament. CAG acts as a guide, friend and philosopher of the Public Accounts Committee of the Parliament.
  • Statement 3 is correct: Information from CAG reports can be used by investigating agencies to press charges against those who have violated the law while managing public finances.
  • Statement 4 is incorrect: While dealing with the audit and accounting of government and PSUs, CAG does not have certain judicial powers for prosecuting those who violate the law. CAG is merely an advisory body and has some quasi-judicial powers.

 

Question 18

Which of the following is/are among the noticeable features of the recommendations of the Thirteenth Finance Commission?

1. A design for the Goods and Services Tax, and a compensation package linked to adherence to the proposed design.

2. A design for the creation of lakhs of jobs in the next ten years in consonance with India’s demographic dividend.

3. Devolution of a specified share of central taxes to local bodies as grants.

Select the correct answer using the codes given below:

(a) 1 only

(b) 2 and 3 only

(c) 1 and 3 only

(d) 1, 2 and 3

Ans: c        

Sub-Theme: Finance Commission

  • Statement 1 is correct: The Thirteenth Finance Commission of India was constituted by the President of India under the chairmanship of Vijay L. Kelkar on 13 November 2007. The 13th FC recommended that both the Centre and the States should conclude ‘Grand Bargain’ to implement the model Goods and Services Act (GST). To incentivize the states, the commission recommended a sanction of the grant of Rs 500 billion.
  • Statement 2 is incorrect: There was no such mention for a design for the creation of lakhs of jobs in the next ten years in consonance with India’s demographic dividend.
  • Statement 3 is correct: State Government should appropriately strengthen their local fund audit departments. As per Article 243I, it mandates the establishment of a Finance Commission in each State to determine the principles on the basis of which adequate financial resources would be ensured for panchayats and municipalities.

 

2011

 

Question 1

In India, if a religious sect/community is given the status of a national minority, what special advantages is it entitled to?

1.It can establish and administer exclusive educational institutions.

2. The President of India automatically nominates a representative of the community to Lok sabha.

3. It can derive benefits from the Prime Minister’s 15-Point Programme.

Which of the statements given above is/are correct?

(a) 1 only

(b) 2 and 3 only

(c) 1 and 3 only

(d) 1, 2 and 3

Ans: c

Sub-Theme: Cultural and educational rights

  • Statement 1 is correct: Though the Constitution of India does not define the term ‘minorities. Article 30 of the Constitution mandates that all minorities have the right to establish and administer educational institutions of their choice.
  • Statement 2 is incorrect: As per the constitution, there is no provision of communal representation in Lok Sabha.
  • Statement 3 is correct: The objective of the Prime Minister’s 15-Point Programme is to enhance opportunities for education and ensure an equitable share for minorities in economic activities and employment, through existing and new schemes, enhanced credit support for self-employment, and recruitment to State and Central Government jobs in those districts which have substantial minority populations.

 

Question 2

Under the constitution of India, which one of the following is not a fundamental duty?

(a) To vote in public elections

(b) To develop the scientific temper

(c) To safeguard public property

(d) To abide by the Constitution and respect its ideals

Ans: a

Sub-Theme: List of Fundamental Duties

Option (a) is correct:

  • Article 51A of part IV-A of the Indian Constitution deals with the Fundamental Duties. These duties are the moral and civic obligations that all citizens of a nation
  • To vote in public elections is not a fundamental duty as it is not included in article 51A.
  • Only the Fundamental Rights, not the Fundamental Duties, were included in the original constitution.
  • In 1976, the Fundamental Duties were first recommended by the Swaran Singh Committee, its need was felt during the internal emergency (1975-77) and were added to the Constitution by 42nd Constitutional Amendment Act, 1976.
  • Later, one more Fundamental Duty was added by 86th Constitutional Amendment Act, 2002.

List of Fundamental Duties:

  • To abide by the Constitution and respect National Flag and National Anthem (Not national song)
  • To follow the noble ideals that inspired the struggle for freedom.
  • To protect the sovereignty, unity and integrity of India.
  • To render national service when called upon to do so.
  • To promote the common spirit of brotherhood amongst all Indians + to renounce practices derogatory to the dignity of women.
  • To preserve the rich heritage of the country’s
  • To protect the national environment including forests, lakes, wildlife, etc.
  • To develop scientific temper, humanism and spirit of reform and inquiry.
  • To safeguard public property and to abjure violence.
  • To strive towards excellence in all spheres of individuals.
  • To provide opportunity for education to his child between the age 6 and 14 years

 

Question 3

The authorization for the withdrawal of funds from the Consolidated Fund of India must come from:

(a) The President of India

(b) The Parliament of India

(c) The Prime Minister of India

(d) The Union Finance Minister

Ans: b

Sub-Theme: Consolidated Fund of India

Consolidated Fund of India (Article 266)

  • All receipts are credited to the Consolidated fund of India, and all payments are debited from it.
  • All the legally authorised payments on behalf of the Government of India are made out of this fund.
  • No money out of this fund can be issued except in accordance with a parliamentary law.

Different Funds with GoI

Consolidated Fund Of India

(Article 266)

Public Account Of India (Article 266) Contingency Fund Of India (Article 267)
•   It is a fund to which all receipts are credited, and all payments are debited. •   All public money other than those which are credited to the CFI shall be credited here. •   Amounts determined by parliament by law are paid from time to time into this fund.
•   All the legally authorised payments on behalf of the Government of India are made out of this fund.

•   No money out of this fund can be issued except in accordance with a parliamentary law.

•   Includes provident fund deposits, judicial deposits, savings bank deposits, departmental deposits, remittances and so on.

•   Operated by executive action.

•   Fund is placed at the disposal of the president, and he can make advances out of it to meet unforeseen expenditure.

•   Held by the finance secretary on behalf of the president. It is operated by executive action.

 

Question 4

All revenues received by the Union Government by way of taxes and other receipts for the conduct of Government business are credited to the:

(a) Contingency Fund of India

(b) Public Account

(c) Consolidated Fund of India

(d) Deposits and Advances Fund

Ans: c

Sub-Theme: Different Funds

Consolidated Fund of India (Article 266)

  • All receipts are credited to the Consolidated fund of India, and all payments are debited from it. Thus, all revenues received by the Union Government by way of taxes and other receipts for the conduct of Government business are credited to it.
  • All the legally authorised payments on behalf of the Government of India are made out of this fund.

Additional Information:

The accounts of Government are kept in three parts: –

  1. Consolidated Fund of India
  2. Contingency Fund of India and
  3. Public Account

 

Question 5

When the annual Union Budget is not passed by the Lok Sabha:

(a) The Budget is modified and presented again

(b) The Budget is referred to the Rajya Sabha for suggestions

(c) The Union Finance Minister is asked to resign

(d) The Prime Minister submits the resignation of Council of Ministers

Ans: d

Sub-Theme: Union Budget/Annual Financial Statement/Provision of Resignation for PM/CoM

According to Article 112 of the Constitution, the Finance Minister must present the Budget or annual financial statement each year. It is a statement of estimated receipts and expenditure of the Government of India that has to be laid before the Parliament in respect of every financial year.

Provision of Resignation for PM/CoM:

  • When the annual Union Budget is not passed by the Lok Sabha, the PM and Council of Ministers submits their resignation.
  • If the money bill has not been passed in Lok Sabha, it leads to the dissolution of Lok Sabha i.e. it questions the majority of the Lok Sabha and therefore the government needs to pass a confidence motion to prove the majority in the house.
  • The PM and the Council of Ministers resign if a Vote of No Confidence has been passed in the Lok Sabha.

 

Question 6

 What is the difference between “vote-on-account” and “interim budget”?

1. The provision of a “vote-on-account” is used by a regular Government, while an “interim budget” is a provision used by a caretaker Government.

2. A “vote-on-account” only deals with the expenditure in the Government’s budget, while an “interim budget” includes both expenditure and receipts.

Which of the statements given above is/are correct?

(a) 1 only

(b) 2 only

(c) Both 1 and 2

(d) Neither 1 nor 2

Ans: b

Sub-Theme: Interim Budget vis-a-vis Vote On Account

Statement 1 is incorrect: During the time of approaching elections, it is not practical to present a full budget, therefore the government presents an interim budget. Interim Budget can be presented by all governments whether incumbent or regular or caretaker.

Statement 2 is correct: Vote-on-Account is a provision by which the government seeks Parliament’s approval for funds that are sufficient to bear the expenditure till the formation of a new government takes place. A vote-on-account contains only the expenditure of the government’s budget while an Interim Budget is a complete set of accounts i.e. it includes both expenditure and receipts.

 

Question 7

The Constitution (Seventy-Third Amendment) Act, 1992, which aims at promoting the Panchayati Raj Institutions in the country, provides for which of the following?

1. Constitution of District Planning Committees.

2. State Election Commissions to conduct all panchayat elections.

3. Establishment of state Finance Commissions.

Select the correct answer using the codes given below:

(a) 1 only

(b) 1 and 2 only

(c) 2 and 3 only

(d) 1, 2 and 3

Ans: d

Sub-Theme: Salient features of 73rd Amendment Act

Statement 1 is correct: The 74th Constitutional Amendment Act provides for a District Planning Committee to consolidate the plans prepared by Panchayats and Municipalities (Article 243ZD) not 73rd Amendment. The 73rd Constitutional Amendment Act, 1992 added a new part IX to the Constitution of India. This part is entitled as “The Panchayats” and contains articles 243 to 243 O.

Statement 2 is correct: According to the 73rd Constitutional Amendment Act, the conduct of Panchayati Raj elections is vested in the state election commission consisting of a state election commissioner appointed by the Governor of the state.

Statement 3 is correct: 73rd Amendment Act also mandates for the establishment of State Finance Commissions.

 

Question 8

Consider the following statements:

In India, a Metropolitan Planning Committee

1. Is constituted under the provisions of the constitution of India.

2. Prepares the draft development plans for the metropolitan area.

3. Has the sole responsibility for implementing Government sponsored schemes in the metropolitan area.

Which of the statements given above is/are correct?

(a) 1 and 2 only

(b) 2 only

(c) 1 and 3 only

(d) 1, 2 and 3

Ans: a

Sub-Theme: Metropolitan Planning Committee

Statement 1 is correct: Article 243 ZE of Part IX A provides for establishment of the Metropolitan  Planning  Committee  (MPC).

The Constitution under Article 243P mandates formation of Metropolitan Planning Committees (MPCs) in all metropolitan areas where population is above 10 Lakh. The elected officials of the municipalities and the heads of the panchayats in the metropolitan area elects 2/3rd of the MPC members from among themselves. 1/3rd is nominated. Elected members are proportionate to the rural: urban population ratio.

Statement 2 is correct: MPC prepares the draft development plans for the metropolitan area. Statement 3 is incorrect: MPC helps in draft planning and not in implementing the Govt sponsored schemes in the metropolitan area.

NOTE: In statement 3, after reading the extreme/absolute phrase “sole”, we can take the calculable risk of marking it as incorrect. This logic is also supported by reasoning that MPC is a “planning body” and statement 3 is talking about “implementation functions”.

 

Question 9

With reference to the Finance Commission of India, which of the following statements is correct?

(a) It encourages the inflow of foreign capital for infrastructure development

(b) It facilitates the proper distribution of finances among the Public Sector Undertakings

(c) It ensures transparency in financial administration

(d) None of the statements (a), (b) and (c) given above is correct in this context

Ans: (d)         

Sub-Theme: Finance Commission

Option (d) is correct: The correct option is ‘(d) None of the statements (a), (b) and (c) given above is correct in this context’ as explained in the previous question.

 Final Result – CIVIL SERVICES EXAMINATION, 2023.   Udaan-Prelims Wallah ( Static ) booklets 2024 released both in english and hindi : Download from Here!     Download UPSC Mains 2023 Question Papers PDF  Free Initiative links -1) Download Prahaar 3.0 for Mains Current Affairs PDF both in English and Hindi 2) Daily Main Answer Writing  , 3) Daily Current Affairs , Editorial Analysis and quiz ,  4) PDF Downloads  UPSC Prelims 2023 Trend Analysis cut-off and answer key

THE MOST
LEARNING PLATFORM

Learn From India's Best Faculty

      

 Final Result – CIVIL SERVICES EXAMINATION, 2023.   Udaan-Prelims Wallah ( Static ) booklets 2024 released both in english and hindi : Download from Here!     Download UPSC Mains 2023 Question Papers PDF  Free Initiative links -1) Download Prahaar 3.0 for Mains Current Affairs PDF both in English and Hindi 2) Daily Main Answer Writing  , 3) Daily Current Affairs , Editorial Analysis and quiz ,  4) PDF Downloads  UPSC Prelims 2023 Trend Analysis cut-off and answer key

Quick Revise Now !
AVAILABLE FOR DOWNLOAD SOON
UDAAN PRELIMS WALLAH
Comprehensive coverage with a concise format
Integration of PYQ within the booklet
Designed as per recent trends of Prelims questions
हिंदी में भी उपलब्ध
Quick Revise Now !
UDAAN PRELIMS WALLAH
Comprehensive coverage with a concise format
Integration of PYQ within the booklet
Designed as per recent trends of Prelims questions
हिंदी में भी उपलब्ध

<div class="new-fform">







    </div>

    Subscribe our Newsletter
    Sign up now for our exclusive newsletter and be the first to know about our latest Initiatives, Quality Content, and much more.
    *Promise! We won't spam you.
    Yes! I want to Subscribe.